You are on page 1of 123

.

Test-26 (Subject)
( Insta Prelims Test Series 2023 ) Total Marks : 200.00

1. Consider the following statements regarding Productivity in an Ecosystem


1. Gross primary productivity of an ecosystem is required by plants in respiration.
2. Species of plants present in a particular area and their photosynthetic capability influences
primary productivity of that area.
3. The annual net primary productivity of oceans is twice that of the land surface.

Which of the statements given above is/are correct?


A. 1 and 2 only
B. 1 only
C. 2 and 3 only
D. 1, 2 and 3

2. Consider the following statements regarding National Clean Air Programme.


1. It is intended to reduce concentration of particulate matter to at least 50% by 2025 taking 2017
as the base year for comparison.
2. Under it, non-attainment cities are identified by the Central Pollution Control Board based on air
quality data from 2014- 2018.
3. It is implemented in non-attainment cities by the Centre for Research on Energy and Clean Air.

Which of the statements given above is/are correct?


A. 1 and 3 only
B. 1 only
C. 2 only
D. 2 and 3 only

3. Consider the following statements regarding Methane


1. Global warming potential of methane is higher than nitrous oxide.
2. A decrease in nitrogen oxide pollution can cause increase in methane concentration in the
atmosphere.
3. ‘Harit Dhara’ portal launched by the government helps to track methane emissions from various
sources.

Which of the statements given above is/are correct?


A. 1 only
B. 1 and 3 only
C. 2 and 3 only
D. 2 only

4. Consider the following statements


1. In an ecosystem, the amount of energy gets reduced as it moves to a successive higher tropic
level from a lower one.
2. Traffic level efficiency for all terrestrial ecosystems is always greater than 50%.

Which of the statements given above is/are correct?

instacourses.insightsonindia.com 1
.
Test-26 (Subject)
( Insta Prelims Test Series 2023 ) Total Marks : 200.00

A. 1 only
B. 2 only
C. Both 1 and 2
D. Neither 1 nor 2

5. The term Trophic cascade refers to

A. Incidence of formation of photochemical smog in the tropical belt.


B. Ecological phenomenon that is triggered by addition or removal of top predators
C. Increasing concentration of major air pollutants in the lower troposphere.
D. None of them above.

6. The Kunming-Montreal Global Biodiversity Framework (GBF) aims to


1. Restore 30% of degraded terrestrial and marine ecosystems by 2030.
2. Halt the risk of extinction of all known species by 2030.
3. Reduce risk from pesticides by at least 30% by 2050.
4. Reduce loss of nutrients to the environment by at least 50% by 2030.

Select the correct answer using the codes given below


A. 1 and 4 only
B. 1, 2 and 3 only
C. 2 and 3 only
D. 3 and 4 only

7. Consider the following statements


1. Melting of glaciers due to climate change can affect plankton growth, which in turn will affect
the top predators in Marine ecosystems.
2. Climate change affects shell formation in shelled Marine organisms.
3. ActNow Campaign has been launched by the United Nations to preserve Marine biodiversity.

Which of the statements given about is/ are correct?


A. 2 and 3 only
B. 1 and 2 only
C. 1 only
D. 1, 2 and 3

8. Consider the following statements regarding System of Air Quality and Weather
Forecasting (SAFAR)
1. It has been introduced by Ministry of Environment, Forest and Climate Change along with Indian
Institute of Tropical Metrology.
2. It monitors the UV and solar radiation along with common air pollutants.
3. It has been recognised by the World Meteorological Organisation as a prototype activity due to
high standards maintained in its maintenance.

instacourses.insightsonindia.com 2
.
Test-26 (Subject)
( Insta Prelims Test Series 2023 ) Total Marks : 200.00

Which of the statements given above is/are correct?


A. 2 and 3 Only
B. 1 only
C. 1 and 3 Only
D. 1, 2 and 3

9. Kelp forests were in news recently; Kelp is

A. Lichens found in marshes


B. Species of Brown Algae
C. Edible mushroom species
D. Freshwater Dinoflagellates

10. The pollutants covered under National Ambient Air Quality Standards include
1. Carbon monoxide
2. Nickel
3. Carbon dioxide
4. Benzopyrene
5. Arsenic
6. Asbestos

Select the correct answer using the codes given below


A. 1, 2, 4, and 5 only
B. 1, 3, 4 and 5 only
C. 1, 2, 3, 4 and 6 only
D. All of the above

11. Consider the following statements regarding bioplastics


1. These are biodegradable materials derived from renewable biomass sources.
2. Microorganisms can convert bioplastics into water and carbon dioxide.
3. Bioplastics made from vegetable oils contain Phthalates and hence are non-biodegradable in
nature.

Which of the statements given above is/are correct?


A. 2 only
B. 2 and 3 only
C. 1 and 2 only
D. 1 only

12. ‘Heimang’ is sometimes mentioned in news; regarding it, consider the following
statements
1. These deciduous plants bear citrus fruits known as Maibis.
2. They are commonly found in north-eastern states of India.
3. The Heimang fruit is useful for treatment of kidney diseases and stomach ulcer.
instacourses.insightsonindia.com 3
.
Test-26 (Subject)
( Insta Prelims Test Series 2023 ) Total Marks : 200.00

Which of the statements given above is/are correct?


A. 1 and 2 only
B. 1 and 3 only
C. 2 and 3 only
D. 1, 2 and 3

13. Consider the following statements regarding Green Urban Oases Programme
1. It is aimed at transforming dryland cities into green urban areas and strengthening their overall
resilience to climatic crises.
2. It has been launched to contribute to the Green Cities Initiative of the Food and Agriculture
Organisation.

Which of the statements given above is/are correct?


A. 1 only
B. 2 only
C. Both 1 and 2
D. Neither 1 nor 2

14. Edge Effect refers to

A. Increase in diversity of species in the zone of transition between two neighbouring


ecosystems
B. The tendency of an ecosystem to maintain its state of stable equilibrium
C. The definite mass of living material present at each tropic level at a particular time.
D. The total range of environmental conditions under which a species can survive.

15. Consider the following statements regarding Keystone species


1. Disappearance of Keystone species from an ecosystem result in introduction of invasive species
in that habitat.
2. They act as popular mascots for conservation issues.
3. Both producers and consumers can be a keystone species for an ecosystem.

Which of the statements given above is/are correct?


A. 1 and 3 only
B. 2 only
C. 2 and 3 only
D. 1 only

16. Which one of the following best describes the term ' Alpha diversity'?

A. Relative abundance of species in terms of number in a region


B. Diversity of species within the same community or habitat
C. Diversity of habitats over the whole geographical area.

instacourses.insightsonindia.com 4
Download From - https://upscmaterial.online/

.
Test-26 (Subject)
( Insta Prelims Test Series 2023 ) Total Marks : 200.00

D. Variety of genes contained in a particular species.

17. Consider the following statements


1. London smog occurs predominantly in urban areas mainly due to emission of carbon monoxide
from vehicles.
2. Los Angeles smog results from a high concentration of Sulphur dioxide in the air.
3. Haze is an atmospheric phenomenon in which dry dust and smoke particles arising from
activities such as farming and wildfires obscure the clarity of the sky, without involving
condensation.

Which of the statements given above is/are correct?


A. 3 only
B. 1 and 3 only
C. 2 only
D. 1 and 2 only

18. Consider the following statements


1. Waste Electrical and Electronic Equipment (WEEE) Forum is a not-for-profit association of
producer responsibility organisations across the world.
2. It adopted the Nairobi declaration to provide innovative solutions for management of electronic
wastes.
3. Karo Sambhav is India's leading Producer Responsibility Organisation that works in collaboration
with WEEE Forum.

Which of the statements given above is/are correct?


A. 2 and 3 only
B. 1 and 2 only
C. 1 and 3 only
D. 1, 2 and 3

19. Consider the following statements regarding the Community Forest Resource (CFR)
Rights:
1. These rights give the authority to the Gram Sabha to adopt local traditional practices of forest
conservation.
2. The CFR rights provide authority over non-timber forest products but do not cover the Nistar
Rights.

Which of the statements given above is/are correct?


A. 1 only
B. 2 only
C. Both 1 and 2
D. Neither 1 nor 2

instacourses.insightsonindia.com 5

https://upscmaterial.online/
Download From - https://upscmaterial.online/

.
Test-26 (Subject)
( Insta Prelims Test Series 2023 ) Total Marks : 200.00

20. The technique of ‘Amine Scrubbing’ is sometimes mentioned in news in the context
of-

A. Water Eutrophication
B. Coral Restoration
C. Carbon Capture and storage
D. Sewage Management

21. Consider the following statements:


1. The Carbon Border Adjustment Mechanism is formulated by India to tax carbon-intensive
products such from 2026.
2. The carbon border tax involves imposing an import duty on a product manufactured in a
country with more lax climate rules than the one buying it.
3. The Carbon Border Adjustment Mechanism is widely criticised for shifting of responsibilities from
developed to developing countries.

Which of the statements given above is/are correct?


A. 1 only
B. 1 and 2 only
C. 2 and 3 only
D. 1, 2 and 3

22. With reference to Coral Bleaching, consider the following statements:


1. It happens when the corals experience high levels of ocean acidity.
2. The El Niño weather pattern acts as a contributing factor to coral bleaching.
3. Reefs once bleached cannot be revived due mainly to the weed settlement.

Which of the statements given above is/are correct?


A. 1 only
B. 1 and 2 only
C. 2 and 3 only
D. 1, 2 and 3

23. Which of the following sub-schemes is/are included under the purview of National
Bioenergy Programme?
1. Biogas Programme
2. Renewable Energy Programme
3. Waste to Energy Programme

Select the correct answer using the code given below:


A. 1 only
B. 1 and 3 only
C. 2 and 3 only
D. 1, 2 and 3

instacourses.insightsonindia.com 6

https://upscmaterial.online/
Download From - https://upscmaterial.online/

.
Test-26 (Subject)
( Insta Prelims Test Series 2023 ) Total Marks : 200.00

24. Consider the following statements regarding the CITES (Convention on International
Trade in Endangered Species of Wild Fauna and Flora):
1. India voluntarily agreed to be bound by this convention when it entered into force.
2. All import and export species covered under CITES must be authorised through a permit
system.
3. CITES Appendix I includes species not threatened with extinction but in trade are strictly
regulated.

Which of the statements given above is/are not correct?


A. 1, 2 and 3
B. 1 and 2 only
C. 3 only
D. 1 only

25. The Adaptation Gap Report 2022 has been recently released by-

A. United Nations Environment Programme


B. Wildlife Trust of India
C. International Union for Conservation of Nature
D. Global Environment Facility

26. Consider the following:


The purpose of Mission LiFE is-

A. To study a potentially habitable environment in the Jezero Crater of the Martian equator.
B. To mobilise Indians and other global citizens to take collective action for preserving the
environment.
C. To build space bricks that can be used to construct building-like structures on Mars.
D. To conserve and sustainably develop the Pantanal which is the world’s largest tropical
wetland.

27. Consider the following statements regarding the Kalanamak Rice:


1. This variety of rice is cultivated only in the Terai region of Uttar Pradesh.
2. This variety is prone to lodging which has badly impacted the grain filling.
3. This rice variety is protected under the Geographical Indication tag system.

This rice variety is protected under the Geographical Indication tag system.
A. 1 only
B. 2 only
C. 2 and 3 only
D. 1 and 3 only

28. Consider the following statements:

instacourses.insightsonindia.com 7

https://upscmaterial.online/
Download From - https://upscmaterial.online/

.
Test-26 (Subject)
( Insta Prelims Test Series 2023 ) Total Marks : 200.00

1. The Mangrove forests grow only at tropical and subtropical latitudes near the equator.
2. The Mangrove trees can sequester up to four times more carbon than tropical rainforests.
3. The MISHTI programme facilitates the mangrove plantation along India’s coastline and on salt
pan lands.

Which of the statements given above is/are correct?


A. 1 only
B. 1 and 2 only
C. 2 and 3 only
D. 1, 2 and 3

29. The ‘SATAT Scheme’ was launched in India with the aim to-

A. To create a carbon neutral Ladakh and put India on the geothermal power map of the
world.
B. To promote solar power as a sustainable way to transition to a carbon-neutral future.
C. To target domestic manufacturing of electrolysers and production of Green Hydrogen.
D. To establish an ecosystem for production of Compressed Bio Gas from biomass sources in
India.

30. Consider the following statements:


1. The Blue Flag is a voluntary tag given to environment-friendly and clean beaches with hygienic
facilities.
2. The Blue Flag certification was originally implemented in India as an environmental awareness
initiative.

Which of the statements given above is/are correct?


A. 1 only
B. 2 only
C. Both 1 and 2
D. Neither 1 nor 2

31. With reference to Glyphosate, consider the following statements:


1. It is a synthetic organophosphate compound used as a broad-spectrum pesticide.
2. In India it is prescribed for use in tea plantations as well as in paddy fields before sowing.

Which of the statements given above is/are correct?


A. 1 only
B. 2 only
C. Both 1 and 2
D. Neither 1 nor 2

32. Consider the following statements regarding the Dark Sky Reserves:

instacourses.insightsonindia.com 8

https://upscmaterial.online/
Download From - https://upscmaterial.online/

.
Test-26 (Subject)
( Insta Prelims Test Series 2023 ) Total Marks : 200.00

1. It is a designation given to a place that has policies in place to ensure that a tract of land has
minimal artificial light interference.
2. The International Dark Sky Association is an inter-governmental organisation that designates
places as International Dark Sky Places.

Which of the statements given above is/are not correct?


A. 1 only
B. 2 only
C. Both 1 and 2
D. Neither 1 nor 2

33. With reference to Ocean Acidification, consider the following statements:


1. It refers to the reduction in pH of the ocean primarily due to the uptake of carbon dioxide from
the atmosphere.
2. Oysters and clams cannot make or maintain their shells because of the acidic ocean water.
3. Acidic-ocean water can weaken the reefs but create optimum conditions for the corals to
multiply at a rapid speed.

Which of the statements given above is/are correct?


A. 1 only
B. 1 and 2 only
C. 1 and 2 only
D. 1, 2 and 3

34. Which one of the following statements is correct about the Rotterdam Convention?

A. It is a multilateral treaty to promote shared responsibilities in relation to importation of


hazardous chemicals.
B. It is an international treaty that seeks to protect human health and the environment from
anthropogenic emissions.
C. It applies to all activities involving exposure of workers to asbestos in the course of work.
D. It regulates transboundary movements of hazardous wastes and ensures that wastes
are disposed in environmentally sound manner.

35. With reference to the Coal Gasification, consider the following statements:
1. It is a technological process that can convert any carbonaceous raw material into fuel gas.
2. Syngas produced in the process is a derivative of hydrogen and does not release carbon
monoxide or carbon dioxide.
3. India’s coal gasification project is aimed at meeting its commitments under CoP-21 Paris
Agreement.

Which of the statements given above is/are correct?


A. 1 only
B. 2 only

instacourses.insightsonindia.com 9

https://upscmaterial.online/
Download From - https://upscmaterial.online/

.
Test-26 (Subject)
( Insta Prelims Test Series 2023 ) Total Marks : 200.00

C. 1, 2 and 3
D. 1 and 3 only

36. Which of the following is/are the advantages of Direct Seeding?


1. Zero Greenhouse Gas emissions
2. Zero plant stress from transplanting
3. Faster maturation of crops
4. Increases income by reducing cost of cultivation

Select the correct answer using the code given below:


A. 1 and 2 only
B. 3 and 4 only
C. 1 and 3 only
D. 2, 3 and 4 only

37. Consider the following statements:


1. The Global Solar Council was established during COP21 to promote the rapid adoption of solar
energy globally.
2. The Global Solar Council is a member of the Marrakech Partnership for Global Climate Action
within the United Nations Framework on Climate Change.

Which of the statements given above is/are correct?


A. 1 only
B. 2 only
C. Both 1 and 2
D. Neither 1 nor 2

38. Which one of the following statements is correct about the Pala Wetland?

A. This wetland is the largest natural wetland in the state of Mizoram.


B. This wetland contains an entire National Park in its low-lying area.
C. The marshy areas in the wetland provide excellent habitat for sambar deer.
D. The wetland is revered by the Mara people and is a source for irrigation.

39. Consider the following statements:


1. India has joined the First Movers Coalition which is aimed at decarbonising the heavy industry
responsible for 30 per cent of global emissions.
2. The First Movers Coalition is led by United Nations Framework Convention on Climate Change.

Which of the statements given above is/are correct?


A. 1 only
B. 2 only
C. Both 1 and 2

instacourses.insightsonindia.com 10

https://upscmaterial.online/
Download From - https://upscmaterial.online/

.
Test-26 (Subject)
( Insta Prelims Test Series 2023 ) Total Marks : 200.00

D. Neither 1 nor 2

40. Consider the following statements regarding the Great Indian Bustards (GIBs):
1. GIBs are considered as the barometers of health of grassland ecosystems.
2. One of the major threats to the GIBs are overhead power transmission lines.
3. GIBs are declared critically endangered by the International Union for Conservation of Nature.

Which of the statements given above is/are correct?


A. 1 and 2 only
B. 2 and 3 only
C. 1 and 3 only
D. 1, 2 and 3

41. The Cloud Forest Assets Programme focuses on-

A. A weather modification technique that improves a cloud’s ability to produce rain by


artificially adding condensation nuclei to the atmosphere.
B. A specific tropical forest ecosystem that offers the potential to be financed as green
infrastructure.
C. A radar network over the cloud burst-prone areas with high-resolution weather
forecasting models.
D. Regeneration and eco-development of degraded forests and adjoining areas on a
watershed basis.

42. With reference to Ganges River dolphin, consider the following statements:
1. They can only live in freshwater and are essentially blind.
2. They cannot breathe in water and come to surface in short durations.
3. The movement of these dolphins is independent of seasonal patterns.

Which of the statements given above is/are correct?


A. 1 only
B. 3 only
C. 2 and 3 only
D. 1 and 2 only

43. ‘SWAS, SAFAL and STAR’ recently talked about in news are associated with-

A. Weather Forecasting Initiatives


B. Air Quality Information Service
C. Categories of Green Crackers
D. Food Safety Management Standards

44. With reference to Eutrophication, consider the following statements:

instacourses.insightsonindia.com 11

https://upscmaterial.online/
Download From - https://upscmaterial.online/

.
Test-26 (Subject)
( Insta Prelims Test Series 2023 ) Total Marks : 200.00

1. It triggers algal growth on the surface of the water body which prevents penetration of sunlight.
2. It inhibits photosynthetic activity in the water body and creates hypoxic for the aquatic
community.

Which of the statements given above is/are correct?


A. 1 only
B. 2 only
C. Both 1 and 2
D. Neither 1 nor 2

45. Consider the following passage:


1. It is a multilateral environmental agreement that regulates the production and consumption of
man-made chemicals referred to as ozone depleting substances (ODS). It phases down the
consumption and production of the different ozone depleting substances in a step-wise manner.

Which one of the following protocols is best described in the passage given above?
A. Montreal Protocol
B. Kyoto Protocol
C. Nagoya Protocol
D. Cartagena Protocol

46. Consider the following statements regarding the Environmental Protection Agency
(EPA):
1. It is an extension of the United Nations Environment Programme to protect environmental
health.
2. Major areas covered by the EPA include wildlife, food safety and nuclear waste.
3. It regulates the manufacturing, processing and distribution of chemicals and other pollutants.

Which of the statements given above is/are not correct?


A. 1 only
B. 3 only
C. 1 and 2 only
D. 2 and 3 only

47. With reference to the Wild Life (Protection) Act of 1972, consider the following
statements:
1. It provides for establishment of sanctuaries and national parks.
2. It provides for state wildlife advisory boards.
3. It provides for judicially imposed penalties for violating the Act.

Which of the statements given above is/are correct?


A. 1 only
B. 1 and 2 only
C. 2 and 3 only
D. 1, 2 and 3
instacourses.insightsonindia.com 12

https://upscmaterial.online/
Download From - https://upscmaterial.online/

.
Test-26 (Subject)
( Insta Prelims Test Series 2023 ) Total Marks : 200.00

48. Consider the following statements regarding the National Green Tribunal (NGT):
1. It is bound to follow procedures laid down in the Code of Civil Procedure.
2. Its order or award is executable by itself as a decree of the Civil Court.
3. The decision of Tribunal by majority is non-binding on the defaulter.

Which of the statements given above is/are correct?


A. 1 only
B. 1 and 2 only
C. 2 and 3 only
D. 1, 2 and 3

49. Consider the following statements regarding Single Use Plastics


1. More than 90% of single use plastic used globally are sourced from fossil fuels.
2. Plastics remain in the environment as microplastics, that tend to bioaccumulate in human body.
3. They can lead to greenhouse gas emissions.

Which of the statements given above is/are correct?


A. 1 and 3 only
B. 1 and 2 only
C. 3 only
D. 1, 2 and 3

50. Consider the following statements regarding Ecological Succession


1. Deforestation can transform a particular seral stage of succession to an earlier stage.
2. Phytoplankton act as pioneers in case of primary succession in water.
3. Hydrarch succession, that takes place in wetter areas leads to creation of mesic conditions.

Which of the statements given above is/are correct?


A. 1 and 3 only
B. 2 only
C. 2 and 3 only
D. 1, 2 and 3

51. Consider the following statements regarding Biodiversity Hotspots


1. United Nations Environment Program designates the Earth’s most endangered terrestrial
ecoregions' as ‘Biodiversity Hotspots’ under the Man and Biosphere Programme.
2. A Biodiversity hotspot should contain at least 1500 species of vascular plants as endemic
species.
3. To be designated as a biodiversity hotspot, a region should have lost at least 70% of its original
natural vegetation.

Which of the statements given above is/are correct?

instacourses.insightsonindia.com 13

https://upscmaterial.online/
Download From - https://upscmaterial.online/

.
Test-26 (Subject)
( Insta Prelims Test Series 2023 ) Total Marks : 200.00

A. 2 and 3 only
B. 2 only
C. 1 and 2 only
D. 1, 2 and 3

52. Consider the following pairs


Convention : Related to
1. Cartagena Protocol : Safe handling of living modified organisms
2. Bishkek Declaration : Conservation of cheetah
3. Nagoya Protocol : Preservation of ozone layer

How many of the pairs given above are correct?


A. None of the above
B. Only one pair
C. Only two pairs
D. All three pairs

53. Consider the following statements


1. Black carbon is emitted through incomplete combustion of fossil fuels
2. Emission of brown carbon due to burning of fossil fuel releases Tarballs.
3. Mangroves and salt marshes are referred to as blue carbon ecosystems.
4. Due to their heat and light absorption properties, both Brown and black carbon impact the
greenhouse balance of the Earth.

Which of the statements given above is/are correct?


A. 2, 3 and 4 only
B. 1 and 3 only
C. 1, 2 and 3 only
D. All of the above

54. Which of the following is/are primary pollutants in the atmosphere?


1. Carbon dioxide
2. Sulphur trioxide
3. Nitrogen dioxide
4. Sulphur dioxide
5. Carbon monoxide
6. Ozone

Select the correct answer using the codes given below


A. 1, 3, 4 and 5 only
B. 2, 4, 5 and 6 only
C. 1, 2, 3 and 5 only
D. All of the above

instacourses.insightsonindia.com 14

https://upscmaterial.online/
Download From - https://upscmaterial.online/

.
Test-26 (Subject)
( Insta Prelims Test Series 2023 ) Total Marks : 200.00

55. Consider the following statements


1. A Conservation Reserve can be declared by State Government in consultation with local
communities for the purpose of protecting its biodiversity.
2. Marine protected areas in which human activities are strictly regulated than surrounding waters
are defined in India according to IUCN guidelines.
3. Wildlife sanctuaries are areas of natural and geomorphological significance where sustainable
livelihood practices of communities living inside are allowed by law.

Which of the statements given above is/are correct?


A. 1 and 3 only
B. 2 only
C. 2 and 3 only
D. 1, 2 and 3

56. Consider the following statements:


1. Carbon sequestration is the process of capturing and storing atmospheric carbon dioxide.
2. Biological Carbon sequestration is a part of enhanced oil recovery also known as tertiary
recovery.
3. In tertiary recovery, the liquid CO2 is injected into the oil-bearing formation to reduce the
viscosity of the oil.

Which of the statements given above is/are correct?


A. 1 only
B. 2 and 3 only
C. 1 and 3 only
D. 1, 2 and 3

57. With reference to the National Initiative on Climate Resilient Agriculture (NICRA),
consider the following statements:
1. The project aims at strategic research on adaptation and mitigation and demonstration of
technologies in farmers’ fields.
2. Under the initiative, Custom hiring centres have been established in the villages to ensure
availability of farm implements for timely operations.

Which of the statements given above is/are correct?


A. 1 only
B. 2 only
C. Both 1 and 2
D. Neither 1 nor 2

58. The main purpose of FAME India Scheme is-

A. To reduce the number of vehicles running on diesel and petrol.


B. To increase private participation in the space sector.

instacourses.insightsonindia.com 15

https://upscmaterial.online/
Download From - https://upscmaterial.online/

.
Test-26 (Subject)
( Insta Prelims Test Series 2023 ) Total Marks : 200.00

C. To provide infrastructural support to semiconductor plants in India.


D. To provide awareness in taking entrepreneurship as a career option.

59. With reference to the Long-Term Ecological Observatories (LTEO), consider the
following statements:
1. In India, this initiative, is a network of government bodies and non-profits working together to
build datasets on critical ecosystems.
2. The LTEO landscapes include Western Ghats and Himalayas but exclude Central India due to
the lack of biodiversity hotspots.

Which of the statements given above is/are correct?


A. 1 only
B. 2 only
C. Both 1 and 2
D. Neither 1 nor 2

60. Consider the following statements regarding National Adaptation Fund for Climate
Change (NAFCC):
1. It is a joint Initiative of India in collaboration with United Nations to support concrete adaptation
activities which mitigate the adverse effects of climate change.
2. Only the projects that are related to agriculture and forestry are eligible for funding under
NAFCC.

Which of the statements given above is/are correct?


A. 1 only
B. 2 only
C. Both 1 and 2
D. Neither 1 nor 2

61. The Bonn Challenge is sometimes seen in news in context of –

A. Restoration of degraded and deforested landscapes


B. Reduction in consumption of ozone-depleting substances
C. Sustainable use of the oceans and marine resources
D. Resurrection biology and reversing extinction in plants

62. With reference to Coastal regulation Zones (CRZ), consider the following statements:
1. It refers to the regions in the proximity of India’s shoreline where industrial projects are not
allowed.
2. The CRZ-1 includes the most ecologically sensitive areas allowing rare public utilities projects in
its proximity.
3. According to the CRZ, 2018 notification, eco-tourism activities are permitted in CRZ-1 regions.

Which of the statements given above is/are correct?

instacourses.insightsonindia.com 16

https://upscmaterial.online/
Download From - https://upscmaterial.online/

.
Test-26 (Subject)
( Insta Prelims Test Series 2023 ) Total Marks : 200.00

A. 1 only
B. 1 and 2 only
C. 2 and 3 only
D. 1, 2 and 3

63. Consider the following statements regarding the Solid Waste Management (SWM)
Rules 2016:
1. As per the rules, waste generator has to segregate solid waste into two streams namely bio-
degradable and non-biodegradable waste.
2. Environment Pollution (Prevention and Control) Authority is mandated to enforce provisions of
the Solid Waste Management Rules in all the states.
3. There is a provision of additional central assistance to States and Union territories for
implementing these rules.

Which of the statements given above is/are correct?


A. 1 only
B. 3 only
C. 1 and 2 only
D. 1, 2 and 3

64. Consider the following statements regarding Compensatory Afforestation


Management and Planning Authority (CAMPA):
1. Works implemented under the CAMPA includes the supply of energy saving devices to the
people living in fringe villages.
2. The Nagar Van Yojana initiated for creation city forests in urban areas over comes under the
purview of CAMPA.

Which of the statements given above is/are correct?


A. 1 only
B. 2 only
C. Both 1 and 2
D. Neither 1 nor 2

65. Consider the following statements regarding the Coalition Against Wildlife Trafficking
(CAWT):
1. It is an initiative of United Nations Environment Programme.
2. India is a member of this Coalition Initiative.
3. It is exclusively an inter-governmental initiative.

Which of the statements given above is/are not correct?


A. 1 only
B. 2 and 3 only
C. 1, 2 and 3
D. 1 and 3 only

instacourses.insightsonindia.com 17

https://upscmaterial.online/
Download From - https://upscmaterial.online/

.
Test-26 (Subject)
( Insta Prelims Test Series 2023 ) Total Marks : 200.00

66. Consider the following statements


1. Electrolysis of water using renewable energy is commonly used for manufacture of green
hydrogen.
2. India’s first pure green hydrogen plant has been commissioned by National Thermal Power
Corporation in Maharashtra.
3. At present 10% of green hydrogen can be blended with Piped Natural Gas in India.

Which of the statements given above is/are correct?


A. 2 only
B. 1 and 3 only
C. 2 and 3 only
D. 1 only

67. Consider the following pairs


Publication : Published by
1. World Social Report : World Bank
2. World Economic : United Nations Situation and Prospects
3. Gender Parity Index : World Economic Forum

How many of the pairs given above are correct?


A. None of the above
B. Only one pair
C. Only two pairs
D. All three pairs

68. The Kerch Strait joins

A. Sea of Azov and Black Sea


B. Mediterranean Sea and Red Sea
C. Aral Sea and Caspian Sea
D. Norwegian Sea and Barents Sea

69. Consider the following statements regarding Trans fatty acids (TFA)
1. Dairy products contain naturally occurring trans fats.
2. TFA increase the number of high-density lipids in human body.
3. Accumulation of TFA in human body is associated with higher risk of development of insulin
resistance.

Which of the statements given above is/are correct?


A. 1 and 2 only
B. 2 only
C. 1 and 3 only
D. 3 only

instacourses.insightsonindia.com 18

https://upscmaterial.online/
Download From - https://upscmaterial.online/

.
Test-26 (Subject)
( Insta Prelims Test Series 2023 ) Total Marks : 200.00

70. Recently, a new species of bamboo dwelling thick-thumbed bat has been reported for
the first time from South Asia, from

A. Arunachal Pradesh
B. Mizoram
C. Meghalaya
D. Assam

71. The Aichi biodiversity targets often talked about in news were established by the-

A. United Nations Convention of Biological Diversity


B. United Nations Biodiversity Conference (COP15)
C. Cancun Adaptation Framework
D. International Union for Conservation of Nature

72. India’s new Draft Arctic Policy aims at-


1. Expanding scientific research
2. Sustainable tourism
3. Oil and gas exploration
4. Waste disposal practices

Which of the statements given above is/are correct?


A. 1 and 4 only
B. 1, 2 and 3 only
C. 1, 2 and 4 only
D. 1, 2, 3 and 4

73. Consider the following statements regarding the Vienna Convention:

1. It speaks to the enormity of ozone depletion and the willingness of countries to work together to
solve it.
2. It essentially requires countries to take control actions to protect the ozone layer.

Which of the statements given above is/are correct?


A. 1 only
B. 2 only
C. Both 1 and 2
D. Neither 1 nor 2

74. With reference to Ramsar Convention on Wetlands, consider the following


statements:
1. It provides single most global framework for intergovernmental cooperation on wetland issues.
2. At the time of joining, each Contracting Party must designate at least one wetland site within

instacourses.insightsonindia.com 19

https://upscmaterial.online/
Download From - https://upscmaterial.online/

.
Test-26 (Subject)
( Insta Prelims Test Series 2023 ) Total Marks : 200.00

their territory as a wetland of International Importance.


3. The Convention's Strategic Plan acknowledges importance of partnerships in implementing the
Convention.

Which of the statements given above is/are correct?


A. 1 only
B. 1 and 2 only
C. 2 and 3 only
D. 1, 2 and 3

75. Which of the following specific areas are encompassed in the missions of National
Action Plan on climate change (NAPCC)?
1. Enhanced Energy Efficiency
2. Deep Sea Mining
3. Green India
4. Sustainable Agriculture

Select the correct answer using the code given below:


A. 1 and 2 only
B. 1, 3 and 4 only
C. 2 and 4 only
D. 1, 2, 3 and 4

76. Which one of the following statements best describes Kigali Agreement?

A. It calls for a gradual reduction in the consumption and production of the


hydrofluorocarbons.
B. It helps countries to achieve sustainable impact on criminal justice and corruption
prevention.
C. It supports parties in the implementation of obligations under Rotterdam and Stockholm
Conventions.
D. It is a standardized way of communicating the environmental impacts of acidification and
eutrophication.

77. Which of the following statements is/are not correct about the International Blue
Carbon Initiative?
1. The Initiative is a global program and works to restore coastal ecosystems for their role in
reducing impacts of global climate change.
2. The Initiative focuses on mangroves, salt marshes and seagrasses which are found on every
continent of the world.

Select the correct answer using the code given below:


A. 1 only
B. 2 only
C. Both 1 and 2
instacourses.insightsonindia.com 20

https://upscmaterial.online/
Download From - https://upscmaterial.online/

.
Test-26 (Subject)
( Insta Prelims Test Series 2023 ) Total Marks : 200.00

D. Neither 1 nor 2

78. With reference to the REDD+, consider the following statements:


1. It is a framework created by the UNFCCC Conference of the Parties.
2. It aims to fulfil the commitment to climate actions in the forest sector.
3. The Parties of the Paris Agreement do not encourage its implementation.

Which of the statements given above is/are correct?


A. 1 only
B. 2 only
C. 1 and 2 only
D. 2 and 3 only

79. Consider the following statements:


1. Any process that uses fossil fuels releases more carbon into the atmosphere than carbon sinks
can absorb.
2. Cattle farming contributes to deforestation but does not contribute in the depletion of carbon
sinks.
3. Both the oceans as well as the forests act as the natural carbon sink in the world.

Which of the statements given above is/are correct?


A. 1 only
B. 2 and 3 only
C. 1 and 3 only
D. 1, 2 and 3

80. Consider the following statements regarding Indian Antarctic Bill, 2022:
1. It prohibits private expeditions to Antarctica without written authorisation by a member country.
2. It extends the jurisdiction of Indian courts to Antarctica but has no penal provision for crimes by
the foreign citizens on the continent.
3. It bars the testing of nuclear devices but has no provisions against disposing waste in
Antarctica.

Which of the statements given above is/are correct?


A. 1 only
B. 1 and 2 only
C. 2 and 3 only
D. 1, 2 and 3

81. Prosopis Juliflora, sometime seen in the news, is native to

A. Mexico
B. Africa

instacourses.insightsonindia.com 21

https://upscmaterial.online/
Download From - https://upscmaterial.online/

.
Test-26 (Subject)
( Insta Prelims Test Series 2023 ) Total Marks : 200.00

C. Asia
D. Australia

82. Which of the following is/are the objectives of National Green Hydrogen Mission
1. Development of green hydrogen production capacity of at least 5 MMT by 2030.
2. Cumulative reduction in fossil fuel imports over Rs. One lakh crore by 2030.

Which of the statements given above is/are correct?


A. 1 only
B. 2 only
C. Both 1 and 2
D. Neither 1 nor 2

83. Consider the following statements


1. Madhya Pradesh has highest leopard population in India
2. Karnataka has highest tiger population in India
3. Assam has highest elephant population in India.

Which of the statements given above is/are not correct?


A. 1 and 2 only
B. 2 and 3 only
C. 1 and 3 only
D. 1, 2 and 3

84. Consider the following statements regarding Climate Action Tracker (CAT)
1. It is an independent scientific project that tracks government climate action and measures it
against the globally agreed Paris Agreement.
2. It covers all the CO2 emitting countries in the world.
3. It has received generous support from foundations and governments.

Which of the statements given above are correct?


A. 1 only
B. 1 and 3 only
C. 3 only
D. 2 and 3 only

85. Consider the following statements


1. A cheetah has a small and well-round head shape whereas a leopard has a more elongated
head with dark blackish spots on the muzzle.
2. A cheetah has more of an amber-coloured eye as opposed to a leopard which has more of a
green-blue coloured eye.
3. Cheetah are taller and slender in build compared to a leopard which are stronger and bulkier.

instacourses.insightsonindia.com 22

https://upscmaterial.online/
Download From - https://upscmaterial.online/

.
Test-26 (Subject)
( Insta Prelims Test Series 2023 ) Total Marks : 200.00

Which of the statements given above is/are correct?


A. 1 and 3 only
B. 1, 2 and 3
C. 2 and 3 only
D. 2 and 3 only

86. Consider the following statements regarding Forest Advisory Committee (FAC)
1. It is a non-statutory body.
2. It comes under Ministry of Tribal Affairs.

Which of the statements given above is/are correct?


A. 1 only
B. 2 only
C. Both 1 and 2
D. Neither 1 nor 2

87. Indian Star Tortoises, sometime seen in the news, is protected under which schedule
of the Wildlife Protection Act, 1972

A. Schedule I
B. Schedule II
C. Schedule III
D. Schedule IV

88. Consider the following statements


1. Nubra Valley is to the north of Leh.
2. Nubra river is the right tributary of Indus.
3. Indus River flows between Ladakh and Karakoram range

Which of the statements given above is/are correct?


A. 1 only
B. 1 and 2 only
C. 3 only
D. 1, 2 and 3

89. Consider the following statements regarding Ammonia


1. It is the basic building block of all nitrogen fertilizer.
2. It has no odour
3. It can be used as a refrigerant gas, to purify water supplies, and in the manufacture of plastics,
explosives, fabrics, pesticides, dyes and other chemicals.

Which of the statements given above is/are correct?


A. 1 and 2 only
B. 2 and 3 only
instacourses.insightsonindia.com 23

https://upscmaterial.online/
Download From - https://upscmaterial.online/

.
Test-26 (Subject)
( Insta Prelims Test Series 2023 ) Total Marks : 200.00

C. 1 and 3 only
D. 1, 2 and 3

90. Access, Watch and Reserve (AWaRe), sometime seen in the news, is related to

A. Wildlife Conservation
B. Access to drinking water and sanitation
C. Antimicrobial Resistance
D. Climate Change Initiatives

91. Which of the following countries is/are the members of Organization of Turkic States
1. Turkey
2. Afghanistan
3. Tajikistan
4. Pakistan

Select the correct answer using the code given below


A. 1 only
B. 2 and 3 only
C. 1 and 3 only
D. 2 and 4 only

92. Consider the following statements


1. The European Parliament represents the citizens of EU countries and is directly elected by
them.
2. The heads of state or government of the EU countries meet, as the European Council, to define
the general political direction and priorities of the European Union.

Which of the statements given above is/are correct?


A. 1 only
B. 2 only
C. Both 1 and 2
D. Neither 1 nor 2

93. Consider the following statements regarding European Investment Bank


1. The Bank borrows money on capital markets and lends it on favourable terms to projects that
support EU objectives.
2. The EIB is the majority shareholder of the European Investment Fund (EIF), which provides
funding to small and medium-sized enterprises (SMEs) through venture capital and risk finance
instruments.

Which of the statements given above is/are correct?


A. 1 only

instacourses.insightsonindia.com 24

https://upscmaterial.online/
Download From - https://upscmaterial.online/

.
Test-26 (Subject)
( Insta Prelims Test Series 2023 ) Total Marks : 200.00

B. 2 only
C. Both 1 and 2
D. Neither 1 nor 2

94. Consider the following statements regarding Schengen area and EU


1. Schengen area allows people and businesses to travel and operate without border checks.
2. It doesn’t include Non – EU countries.
3. The EU budget is funded by a percentage of each member country’s gross national income.

Which of the statements given above is/are correct?


A. 3 only
B. 2 and 3 only
C. 1 and 3 only
D. 1, 2 and 3

95. Which of the following places is/are located in Turkey?


1. Malatya
2. Ekinozu
3. Hama

Select the correct answer using the code given below


A. 1 and 3 only
B. 2 only
C. 1 and 2 only
D. 3 only

96. Burkina Faso is surrounded by which of the following countries?


1. Mali
2. Niger
3. Nigeria
4. Mauritania

Select the correct answer using the code given below:


A. 1 and 3 only
B. 2, 3 and 4 only
C. 1 and 2 only
D. 1, 2, 3 and 4

97. Consider the following statements regarding Boards of Directors


1. The President of the World Bank Group serves as Chairman of the Board, and is selected by the
Executive Directors.
2. The Board usually meets twice a week on Tuesdays and Thursdays.
3. The voting power of each Member country is based on the number of shares it holds.

instacourses.insightsonindia.com 25

https://upscmaterial.online/
Download From - https://upscmaterial.online/

.
Test-26 (Subject)
( Insta Prelims Test Series 2023 ) Total Marks : 200.00

Which of the statements given above is/are correct?


A. 1 only
B. 2 and 3 only
C. 3 only
D. 1, 2 and 3

98. Pink River Dolphin, sometime seen in the news, can be seen in which of the following
river?

A. Amazon
B. Nile
C. Ganga
D. Mekong

99. Consider the following statements regarding National Institute of Solar Energy (NISE)
1. NISE is an autonomous specialized institute under the Ministry of New and Renewable Energy
2. NISE is a technical hub for all solar linked activities, standardization, designing, consultancy,
and skill development programs.

Which of the statements given above is/are correct?


A. 1 only
B. 2 only
C. Both 1 and 2
D. Neither 1 nor 2

100. Greater one-horned rhino, is not naturally found in

A. Pakistan
B. Nepal
C. Bhutan
D. None of the above

instacourses.insightsonindia.com 26

https://upscmaterial.online/
Download From - https://upscmaterial.online/

.
Total Marks : 200.00
Test-26 (Subject)
( Insta Prelims Test Series 2023 )

1. Consider the following statements regarding Productivity in an Ecosystem


1. Gross primary productivity of an ecosystem is required by plants in respiration.
2. Species of plants present in a particular area and their photosynthetic capability influences
primary productivity of that area.
3. The annual net primary productivity of oceans is twice that of the land surface.

Which of the statements given above is/are correct?


A. 1 and 2 only
B. 1 only
C. 2 and 3 only
D. 1, 2 and 3

Correct Answer : A

Answer Justification :

The rate of biomass production in an ecosystem is called productivity.

It can be divided into gross primary productivity and net primary productivity.

Gross primary Productivity of an ecosystem is the rate of production of organic matter


during photosynthesis.

A considerable amount of gross primary productivity is utilised by plants in


respiration.

Hence statement 1 is correct.

Primary productivity depends on the plant species inhabiting a particular area.

It also depends on variety of environment factors, availability of nutrients and


photosynthetic capacity of plants

Hence, it varies for different types of ecosystems.

Hence statement 2 is correct.

The annual net primary productivity of the whole biosphere is approximately


170 billion tons (dry weight) of organic matter

instacourses.insightsonindia.com 1

https://upscmaterial.online/
Download From - https://upscmaterial.online/

.
Total Marks : 200.00
Test-26 (Subject)
( Insta Prelims Test Series 2023 )

of this, despite occupying 70% of the surface, productivity of oceans is only 55


billion tons and the rest is provided by land surface.

Hence statement 3 is incorrect.

2. Consider the following statements regarding National Clean Air Programme.


1. It is intended to reduce concentration of particulate matter to at least 50% by 2025 taking 2017
as the base year for comparison.
2. Under it, non-attainment cities are identified by the Central Pollution Control Board based on air
quality data from 2014- 2018.
3. It is implemented in non-attainment cities by the Centre for Research on Energy and Clean Air.

Which of the statements given above is/are correct?


A. 1 and 3 only
B. 1 only
C. 2 only
D. 2 and 3 only

Correct Answer : C

Answer Justification :

The National Clean Air Programme was launched by the Ministry of Environment, Forest
and Climate Change in 2019 to prepare clean action plans for the country

The goal of NCAP (annual average ambient air quality standard at all locations in the
country) includes 20 to 30% reduction of PM 2.5 and PM 10 concentration in the next 5
years by taking 2017 as the base year for comparison of concentration.

However, recently the centre has set a new target of 40% reduction in particulate
matter concentration by 2026.

Hence statement 1 is incorrect.

Under NCAP, 132 non-attainment cities have been identified across the country based on
air quality data from 2014-2018.

Non-attainment cities are those that have fallen short of the National Ambient Air Quality
Standards over 5 years.

instacourses.insightsonindia.com 2

https://upscmaterial.online/
Download From - https://upscmaterial.online/

.
Total Marks : 200.00
Test-26 (Subject)
( Insta Prelims Test Series 2023 )

NAAQs are notified by the Central Pollution Control Board.

The non-attainment cities are identified by CPCB.

Hence statement 2 is correct.

The NCAP provides for implementation of the city specific action plans to be regularly
monitored by committees at the Central and state levels, namely Steering Committee,
Monitoring Committee and Implementation Committee.

The Centre for Research on Energy and Clean Air (CREA) provided an analysis of the four
year performance of NCAP; according to it, only 38 of the cities achieved their annual
pollution reduction targets.

The CREA is responsible for analysing the program and providing guidance for
management of air pollution in cities.

Hence statement 3 is incorrect.

3. Consider the following statements regarding Methane


1. Global warming potential of methane is higher than nitrous oxide.
2. A decrease in nitrogen oxide pollution can cause increase in methane concentration in the
atmosphere.
3. ‘Harit Dhara’ portal launched by the government helps to track methane emissions from various
sources.

Which of the statements given above is/are correct?


A. 1 only
B. 1 and 3 only
C. 2 and 3 only
D. 2 only

Correct Answer : D

Answer Justification :

According to the IPCC second assessment report the global warming potential of methane
over a period of 100 years is 21 while that of nitrous oxide is 310.

Methane has more than 80 times warming power of carbon dioxide over the first 20 years

instacourses.insightsonindia.com 3

https://upscmaterial.online/
Download From - https://upscmaterial.online/

.
Total Marks : 200.00
Test-26 (Subject)
( Insta Prelims Test Series 2023 )

of its lifetime in the atmosphere but its global warming potential is much lower than that
of nitrous oxide.

Hence statement 1 is incorrect.

Nitrogen oxide levels in the atmosphere influences levels of methane

Nitrogen oxide enters the atmosphere from vehicle exhaust and electrical power
generation plants.

In the troposphere, nitrogen oxide combines with ozone to form hydroxyl radicals; these
radicals in turn remove 85% of the methane annually from the atmosphere.

Hence statement 2 is correct.

The Harit Dhara is an anti-methanogenic feed supplement developed by Indian


Council of Agricultural Research

It is intended to cut down methane emissions by cattle by at least 17 to 20% and also
result in higher milk production.

Hence statement 3 is incorrect.

4. Consider the following statements


1. In an ecosystem, the amount of energy gets reduced as it moves to a successive higher tropic
level from a lower one.
2. Traffic level efficiency for all terrestrial ecosystems is always greater than 50%.

Which of the statements given above is/are correct?


A. 1 only
B. 2 only
C. Both 1 and 2
D. Neither 1 nor 2

Correct Answer : A

Answer Justification :

Energy flow is a fundamental process common to all ecosystem

instacourses.insightsonindia.com 4

https://upscmaterial.online/
Download From - https://upscmaterial.online/

.
Total Marks : 200.00
Test-26 (Subject)
( Insta Prelims Test Series 2023 )

the energy flow through an ecosystem begins with capturing of sunlight by autotrophs
during photosynthesis.

This energy flow is unidirectional that is it flows from a lower to a successively higher
trophic level

As energy flows across trophic levels, it is degraded and less and less is available for the
next trophic level.

Hence statement 1 is correct.

Energy flow across trophic levels is measured as Trophic level efficiency.

Trophic level efficiency is defined as the ratio between production of one trophic level to
the production of next lower trophic level.

This efficiency is never very high

Usually, 90% or even much more than that of energy transferred between trophic levels is
lost as heat

Thus, trophic level efficiency is about 10% or less for most ecosystems.

Hence statement 2 is incorrect.

5. The term Trophic cascade refers to

A. Incidence of formation of photochemical smog in the tropical belt.


B. Ecological phenomenon that is triggered by addition or removal of top predators
C. Increasing concentration of major air pollutants in the lower troposphere.
D. None of them above.

Correct Answer : B

Answer Justification :

Trophic cascade is an ecological phenomenon which is triggered by addition or


removal of top predators in a ecosystem.

instacourses.insightsonindia.com 5

https://upscmaterial.online/
Download From - https://upscmaterial.online/

.
Total Marks : 200.00
Test-26 (Subject)
( Insta Prelims Test Series 2023 )

This may involve significant changes in the relative populations of Predators and prey in a
food chain which often results in dramatic changes in ecosystem structure and nutrient
cycling.

In a three-level food chain and increase or decrease in carnivores causes a decrease or


increase in primary producers such as plants and phytoplankton.

Certain experiments showed that tropic cascades-controlled biomass and production of


phytoplankton, recycling rates of nutrients, the ratio of Nitrogen to Phosphorus available
to phytoplankton, activity of bacteria and sedimentation rates in aquatic ecosystems.

Hence statement B is correct.

6. The Kunming-Montreal Global Biodiversity Framework (GBF) aims to


1. Restore 30% of degraded terrestrial and marine ecosystems by 2030.
2. Halt the risk of extinction of all known species by 2030.
3. Reduce risk from pesticides by at least 30% by 2050.
4. Reduce loss of nutrients to the environment by at least 50% by 2030.

Select the correct answer using the codes given below


A. 1 and 4 only
B. 1, 2 and 3 only
C. 2 and 3 only
D. 3 and 4 only

Correct Answer : A

Answer Justification :

The Kunming-Montreal Global biodiversity framework was adopted at the 15th Conference of
parties to the United Nations Convention of Biological Diversity.

It includes 4 goals and 23 targets for achievement by 2030. The key targets of GBF include:

Restore 30% degraded ecosystems (on land and sea) worldwide by 2030.

Conserve and manage 30% terrestrial, inland water and coastal and Marine areas by 2030

These are included within the 30×30 Deal.

Other targets include

instacourses.insightsonindia.com 6

https://upscmaterial.online/
Download From - https://upscmaterial.online/

.
Total Marks : 200.00
Test-26 (Subject)
( Insta Prelims Test Series 2023 )

Stop the extinction of known species and by 2050, reduce the extinction risk and rate of
all species by tenfold.

Reduce pollution risk and negative impacts of pollution from all sources by 2030 to levels
that are not very harmful to biodiversity and ecosystems

Reduce risk from pesticides by at least 50% by 2030

Reduce nutrients lost to the environment by at least 50% by 2030

Reduce global footprint of consumption by 2030 including reducing overconsumption and


waste generation and decreasing food waste

Sustainable management of areas under agricultural, aquaculture, fisheries and forestry

Reduce the rate of introduction and establishment of invasive alien species by at least
50% by 2030.

Secure the safe, legal and sustainable use and trade of wild species by 2030.

Hence option A is correct.

7. Consider the following statements


1. Melting of glaciers due to climate change can affect plankton growth, which in turn will affect
the top predators in Marine ecosystems.
2. Climate change affects shell formation in shelled Marine organisms.
3. ActNow Campaign has been launched by the United Nations to preserve Marine biodiversity.

Which of the statements given about is/ are correct?


A. 2 and 3 only
B. 1 and 2 only
C. 1 only
D. 1, 2 and 3

Correct Answer : B

Answer Justification :

Climate change in recent times adversely impacts marine ecosystems across the globe.

instacourses.insightsonindia.com 7

https://upscmaterial.online/
Download From - https://upscmaterial.online/

.
Total Marks : 200.00
Test-26 (Subject)
( Insta Prelims Test Series 2023 )

Melting of glaciers due to warmer temperatures and increased rainfall will lead to rise in
levels of freshwater which can affect plankton growth.

The growth of planktons in turn may affect the whole food chain at the top of which lies
predators like whales and dolphins.

Hence statement 1 is correct.

Climate change may lead to decrease in concentration of calcium carbonate which may
affect shell formation in certain marine organisms such as molluscs, krill, etc.

Hence statement 2 is correct

ActNow campaign has been launched by United Nations for individual action on
climate change and sustainability; not particularly for protection of marine
biodiversity.

Hence statement 3 is incorrect.

8. Consider the following statements regarding System of Air Quality and Weather
Forecasting (SAFAR)
1. It has been introduced by Ministry of Environment, Forest and Climate Change along with Indian
Institute of Tropical Metrology.
2. It monitors the UV and solar radiation along with common air pollutants.
3. It has been recognised by the World Meteorological Organisation as a prototype activity due to
high standards maintained in its maintenance.

Which of the statements given above is/are correct?


A. 2 and 3 Only
B. 1 only
C. 1 and 3 Only
D. 1, 2 and 3

Correct Answer : A

Answer Justification :

The System of Air Quality and Weather Forecasting initiative has been introduced by
Ministry of Earth Sciences.

instacourses.insightsonindia.com 8

https://upscmaterial.online/
Download From - https://upscmaterial.online/

.
Total Marks : 200.00
Test-26 (Subject)
( Insta Prelims Test Series 2023 )

It has been developed indigenously by Indian Institute of Tropical Metrology, Pune and is
operationalized by Indian Meteorological Department.

Hence statement 1 is incorrect.

The system monitors regular air quality parameters like PM 2.5, PM 10, nitrogen oxides,
carbon monoxide, Sulphur Dioxide, ozone and will also monitor the existence of
substances like benzene, toluene and xylene.

It also monitors all weather parameters like temperature, rainfall, humidity, wind speed
and wind direction, UV radiation and solar radiation.

Hence statement 2 is correct.

Under the system, based on air quality index on a particular day, health advisory and
related precautions will be forwarded to citizens to prepare them in advance.

SAFAR will accelerate public awareness and preparedness of air pollution and weather
extremes.

The World Meteorological Organisation has recognised SAFAR as a prototype activity on


the basis of high-quality control and Standards maintained in its implementation.

Hence statement 3 is correct.

9. Kelp forests were in news recently; Kelp is

A. Lichens found in marshes


B. Species of Brown Algae
C. Edible mushroom species
D. Freshwater Dinoflagellates

Correct Answer : B

Answer Justification :

Kelp are large brown algae that lives in cool relatively shallow waters close to
the shore.

instacourses.insightsonindia.com 9

https://upscmaterial.online/
Download From - https://upscmaterial.online/

.
Total Marks : 200.00
Test-26 (Subject)
( Insta Prelims Test Series 2023 )

Kelp forests are underwater ecosystems found in shallow water; these are formed by
dense growth of several species.

Kelp forest are always coastal and require shallow relatively clear water.

They remain attached to the sea floor and eventually grow to the surface of the water

They depend on sunlight to produce food and energy.

They provide food for a variety of marine species as well as shelter to various
invertebrates, fishes and other algae.

They provide certain ecosystem services such as coastal protection and carbon
sequestration.

Hence option B is correct

10. The pollutants covered under National Ambient Air Quality Standards include
1. Carbon monoxide
2. Nickel
3. Carbon dioxide
4. Benzopyrene
5. Arsenic
6. Asbestos

Select the correct answer using the codes given below


A. 1, 2, 4, and 5 only
B. 1, 3, 4 and 5 only
C. 1, 2, 3, 4 and 6 only
D. All of the above

Correct Answer : A

Answer Justification :

The National Ambient Air Quality Standards are the standards for ambient air quality in
India

It has been set up by the Central Pollution Control Board under the Air (Prevention and
control of pollution) act 1981.

instacourses.insightsonindia.com 10

https://upscmaterial.online/
Download From - https://upscmaterial.online/

.
Total Marks : 200.00
Test-26 (Subject)
( Insta Prelims Test Series 2023 )

The standards are essential for the development of effective management of ambient air
quality throughout the country.

The latest version of the NAAQs was done in 2009 and at present this version is followed.

The 2009 version lowered the maximum permissible limits for pollutants and made the
standards uniform across the country; earlier, industrial zones had less stringent norms as
compared to residential areas.

Compliance of NAAQs is monitor under the National Air Quality Monitoring Programme,
which is implemented by CPCB.

The ambient air quality standards includes 12 pollutants.

They are Sulphur dioxide, Nitrogen dioxide, Particulate matter ( 10 and 2.5),
ozone, Lead, Arsenic, Nickel, Benzene, Ammonia, Carbon monoxide and
Benzopyrene.

Hence option A is correct.

11. Consider the following statements regarding bioplastics


1. These are biodegradable materials derived from renewable biomass sources.
2. Microorganisms can convert bioplastics into water and carbon dioxide.
3. Bioplastics made from vegetable oils contain Phthalates and hence are non-biodegradable in
nature.

Which of the statements given above is/are correct?


A. 2 only
B. 2 and 3 only
C. 1 and 2 only
D. 1 only

Correct Answer : C

Answer Justification :

Bioplastics are derived from renewable Biomass sources such as vegetable fat and oils,
corn starch, food waste, or from sugar derivatives including starch, cellulose, lactic acid,
etc.

instacourses.insightsonindia.com 11

https://upscmaterial.online/
Download From - https://upscmaterial.online/

.
Total Marks : 200.00
Test-26 (Subject)
( Insta Prelims Test Series 2023 )

These can also be made from agricultural by products.

They can be used to reduce the problem of contaminating plastic waste that is suffocating
the planet and contaminating the environment.

Hence statement 1 is correct.

Bioplastics can be converted into natural substances like water, carbon dioxide and
compost by the action of microorganisms present in the environment.

Hence statement 2 is correct.

Phthalates are a group of chemicals used to make plastics more durable.

Bioplastics are devoid of phthalates.

Bioplastics do not contain additives that are harmful to health including Phthalates and
bisphenol A.

However, not all bioplastics are degradable.

Hence statement 3 is incorrect.

12. ‘Heimang’ is sometimes mentioned in news; regarding it, consider the following
statements
1. These deciduous plants bear citrus fruits known as Maibis.
2. They are commonly found in north-eastern states of India.
3. The Heimang fruit is useful for treatment of kidney diseases and stomach ulcer.

Which of the statements given above is/are correct?


A. 1 and 2 only
B. 1 and 3 only
C. 2 and 3 only
D. 1, 2 and 3

Correct Answer : C

Answer Justification :

instacourses.insightsonindia.com 12

https://upscmaterial.online/
Download From - https://upscmaterial.online/

.
Total Marks : 200.00
Test-26 (Subject)
( Insta Prelims Test Series 2023 )

Heimang plant bears creamy white flowers and spherical shaped fruit the fruits are tiny
but pact with nutrients such as polyphenols, flavonoids and antioxidants.

The flowers stand out against the green leaves of the deciduous tree.

By November and December, the flowers transform into deep red cherry like fruits that
have been used as food and medicine for centuries.

Maibis or Maibas are traditional healers of Manipur.

Hence statement 1 is incorrect.

Heimang widely grows across Manipur and other north-eastern states of India.

According to Maibis, Heimang can be used for common gastrointestinal problems like
diarrhoea and dysentery

They also recommend eating water-soaked foot for injection and stomach ulcer.

They also say that the fruit is useful in treatment of kidney diseases and urinary stones.

Hence statements 2 and 3 are correct.

13. Consider the following statements regarding Green Urban Oases Programme
1. It is aimed at transforming dryland cities into green urban areas and strengthening their overall
resilience to climatic crises.
2. It has been launched to contribute to the Green Cities Initiative of the Food and Agriculture
Organisation.

Which of the statements given above is/are correct?


A. 1 only
B. 2 only
C. Both 1 and 2
D. Neither 1 nor 2

Correct Answer : C

Answer Justification :

instacourses.insightsonindia.com 13

https://upscmaterial.online/
Download From - https://upscmaterial.online/

.
Total Marks : 200.00
Test-26 (Subject)
( Insta Prelims Test Series 2023 )

The Green Urban Oases program was launched by Food and Agriculture
Organisation in 2021.

It aims to improve the resilience of dryland cities by tackling climate, health, food and
economic challenges.

The intention is to transform dryland cities into Green Urban Oases by strengthening their
overall resilience to climatic, health, food and economic crises for the improved health
and well-being of urban communities.

It focuses on developing policy, technical capacity and outline several pathways for
transforming urban areas by planting trees.

The program contributes to the FAO Green Cities initiative that was launched in 2020.

Hence Statements 1 and 2 are correct.

14. Edge Effect refers to

A. Increase in diversity of species in the zone of transition between two neighbouring


ecosystems
B. The tendency of an ecosystem to maintain its state of stable equilibrium
C. The definite mass of living material present at each tropic level at a particular time.
D. The total range of environmental conditions under which a species can survive.

Correct Answer : A

Answer Justification :

Two adjacent ecosystems do not have a clearly demarcating boundaries; rather the
intermediate region is marked by an ecotone.

Ecotone is defined as the zone of transition between two neighbouring ecosystems.

There are changes occurring in community structures at the ecotones.

The tendency for increased variety and diversity of species in this zone of transition
between two neighbouring ecosystems is known as Edge effect

instacourses.insightsonindia.com 14

https://upscmaterial.online/
Download From - https://upscmaterial.online/

.
Total Marks : 200.00
Test-26 (Subject)
( Insta Prelims Test Series 2023 )

For example, in an ecotone between a forest and a grassland, there may be a few types
of grasses, shrubs and stunted trees.

Hence statement a is correct.

The tendency of an ecosystem to maintain its state of stable equilibrium is called


homeostasis.

Hence statement b is incorrect.

Each tropic level has a certain mass of living material at a particular time this is known as
standing crop.

The standing crop is measured as the mass of living organisms or the number in a unit
area.

Hence statement c is incorrect.

Fundamental niche is defined as the total range of environmental conditions under


which a species can survive and reproduce.

Hence statement d is incorrect.

15. Consider the following statements regarding Keystone species


1. Disappearance of Keystone species from an ecosystem result in introduction of invasive species
in that habitat.
2. They act as popular mascots for conservation issues.
3. Both producers and consumers can be a keystone species for an ecosystem.

Which of the statements given above is/are correct?


A. 1 and 3 only
B. 2 only
C. 2 and 3 only
D. 1 only

Correct Answer : A

Answer Justification :

instacourses.insightsonindia.com 15

https://upscmaterial.online/
Download From - https://upscmaterial.online/

.
Total Marks : 200.00
Test-26 (Subject)
( Insta Prelims Test Series 2023 )

A keystone species is an organism that helps define an entire ecosystem

Without this species the ecosystem would be dramatically different or cease to exist
altogether

The species have low functional redundancy that is, if the species were to disappear from
the ecosystem, no other species would be able to fill its ecological niche.

The ecosystem would be forced to radically change allowing new and possibly invasive
species to populate the habitat.

Hence statement 1 is correct.

Flagship species serve as popular Mascot for conservation issues.

A flagship species is one that is chosen to raise support for biodiversity conservation in a
given place or social context.

Hence statement 2 is incorrect.

In a given ecosystem, any organism from plants to fungi may be a keystone


species

They are not always required to be the largest almost abundant species.

Hence statement 3 is correct.

16. Which one of the following best describes the term ' Alpha diversity'?

A. Relative abundance of species in terms of number in a region


B. Diversity of species within the same community or habitat
C. Diversity of habitats over the whole geographical area.
D. Variety of genes contained in a particular species.

Correct Answer : B

Answer Justification :

instacourses.insightsonindia.com 16

https://upscmaterial.online/
Download From - https://upscmaterial.online/

.
Total Marks : 200.00
Test-26 (Subject)
( Insta Prelims Test Series 2023 )

Species evenness is defined as the relative abundance ( in terms of numbers) of species


in a region.

Hence statement a is incorrect.

Alpha diversity refers to diversity of species within the same community or


habitat.

Classification of biodiversity based upon community as a reference was given by


Whittaker in 1960.

Based on this, biodiversity can be classified as Alpha diversity, Beta diversity and Gamma
diversity.

Hence statement b is correct.

Gamma diversity refers to diversity of habitats over the whole geographical area.

Hence statement c is incorrect.

Genetic diversity refers to the variety of jeans contained in a particular species or in all
living species on the planet.

It contains different genetic characteristics of species

it serves as a way for populations to adapt to changing environments.

Hence statement d is incorrect.

17. Consider the following statements


1. London smog occurs predominantly in urban areas mainly due to emission of carbon monoxide
from vehicles.
2. Los Angeles smog results from a high concentration of Sulphur dioxide in the air.
3. Haze is an atmospheric phenomenon in which dry dust and smoke particles arising from
activities such as farming and wildfires obscure the clarity of the sky, without involving
condensation.

Which of the statements given above is/are correct?

instacourses.insightsonindia.com 17

https://upscmaterial.online/
Download From - https://upscmaterial.online/

.
Total Marks : 200.00
Test-26 (Subject)
( Insta Prelims Test Series 2023 )

A. 3 only
B. 1 and 3 only
C. 2 only
D. 1 and 2 only

Correct Answer : A

Answer Justification :

Sulfurous smog is also called London smog.

Sulfurous smog results from a high concentration of Sulphur dioxide in the air which is
caused by the use of sulphur bearing fossil fuels, particularly coal.

Coal was the main source of power in London during the 19th Century; the effects of coal
burning were observed in early 20th century.

Hence statement 1 is incorrect.

Los Angeles smog is actually photochemical smog that occurs predominantly in


urban areas having large number of automobiles.

In this case, nitrogen oxides are the primary emissions.

Photochemical (summer smog) forms when pollutants that has nitrogen oxides and
organic compounds react together in the presence of Sunlight and forms ozone.

The resulting smog causes a brownish coloration of the atmosphere with reduced visibility
along with causing damage to plants as well as human health.

Hence statement 2 is incorrect.

Haze is traditionally an atmospheric phenomenon where dust, smoke and other


dry particles obscure the clarity of the sky

Here no condensation is involved

Sources for dust particles include farming (ploughing in dry weather), traffic, industry and
Wildfires.

instacourses.insightsonindia.com 18

https://upscmaterial.online/
Download From - https://upscmaterial.online/

.
Total Marks : 200.00
Test-26 (Subject)
( Insta Prelims Test Series 2023 )

Hence statement 3 is correct.

18. Consider the following statements


1. Waste Electrical and Electronic Equipment (WEEE) Forum is a not-for-profit association of
producer responsibility organisations across the world.
2. It adopted the Nairobi declaration to provide innovative solutions for management of electronic
wastes.
3. Karo Sambhav is India's leading Producer Responsibility Organisation that works in collaboration
with WEEE Forum.

Which of the statements given above is/are correct?


A. 2 and 3 only
B. 1 and 2 only
C. 1 and 3 only
D. 1, 2 and 3

Correct Answer : C

Answer Justification :

Waste Electrical and Electronic Equipment (WEEE) Forum is a not-for-profit


association of 46 producer responsibility organisations across the world.

It was founded in April 2002.

Through exchange of best practices and access to its reputable knowledge base toolbox,
the Forum enables its members to improve their operations and be known as promoters
of the circular economy.

Hence statement 1 is correct.

The Basel convention was adopted in 1989 by the conference of Plenipotentiaries in


Basel, Switzerland regarding control of Trans boundary movement of hazardous waste
and their disposal it came into force in 1992.

The original convention did not mention about electronic waste but later it addressed the
issue of e-waste in 2006 at conference of parties (COP8)

The Nairobi declaration was adopted at COP9 of the Basel convention and aimed
at creating innovative solutions for environmentally sound management of

instacourses.insightsonindia.com 19

https://upscmaterial.online/
Download From - https://upscmaterial.online/

.
Total Marks : 200.00
Test-26 (Subject)
( Insta Prelims Test Series 2023 )

electronic Wastes.

Hence statement 2 is incorrect.

Karo Sambhav is India's leading Producer Responsibility Organisation.

It was founded in 2017 and is currently spread across 29 States, 3 union territories and
over 60 cities.

It joined WEEE Forum in January 2020.

It has recycled/processed more than 16000 terms of electronic waste on behalf of 40 +


electronic producers.

Hence statement 3 is correct.

19. Consider the following statements regarding the Community Forest Resource (CFR)
Rights:
1. These rights give the authority to the Gram Sabha to adopt local traditional practices of forest
conservation.
2. The CFR rights provide authority over non-timber forest products but do not cover the Nistar
Rights.

Which of the statements given above is/are correct?


A. 1 only
B. 2 only
C. Both 1 and 2
D. Neither 1 nor 2

Correct Answer : A

Answer Justification :

Explanation:

The Community Forest Resource rights under Section 3(1)(i) of the Scheduled Tribes and Other
Traditional Forest Dwellers (Recognition of Forest Rights) Act (commonly referred to as the
Forest Rights Act or the FRA) provide for recognition of the right to “protect, regenerate or
conserve or manage” the community forest resource.

Statement 1 is correct: These rights allow the community to formulate rules for forest use by

instacourses.insightsonindia.com 20

https://upscmaterial.online/
Download From - https://upscmaterial.online/

.
Total Marks : 200.00
Test-26 (Subject)
( Insta Prelims Test Series 2023 )

itself and others and thereby discharge its responsibilities under Section 5 of the FRA.

Statement 2 is not correct: CFR rights, along with Community Rights (CRs) under Sections
3(1)(b) and 3(1)(c), which include nistar rights and rights over non-timber forest products,
ensure sustainable livelihoods of the community.

These rights give the authority to the Gram Sabha to adopt local traditional practices of forest
conservation and management within the community forest resource boundary.

Kanger Ghati National Park is the second national park, after Simlipal in Odisha, where CFR
rights have been recognised.

Source:
https://indianexpress.com/article/explained/explained-what-are-community-forest-rig
hts-why-do-they-
matter-7939921/#:~:text=The%20community%20forest%20resource%20area%20is%
20the%20common,use%20of%20landscape%20in%20case%20of%20pastoralist%20co
mmunities.

20. The technique of ‘Amine Scrubbing’ is sometimes mentioned in news in the context
of-

A. Water Eutrophication
B. Coral Restoration
C. Carbon Capture and storage
D. Sewage Management

Correct Answer : C

Answer Justification :

Explanation:

Option (c) is correct: Carbon capture and storage (CCS) refers to a collection of technologies
that can combat climate change by reducing carbon dioxide (CO2) emissions.

Amine scrubbing has been used to separate carbon dioxide (CO2) from natural gas and
hydrogen since 1930.

It is a robust technology and is ready to be tested and used on a larger scale for CO2 capture
from coal-fired power plants.

The minimum work requirement to separate CO2 from coal-fired flue gas and compress CO2 to
150 bar is 0.11 megawatt-hours per metric ton of CO2.

Process and solvent improvements should reduce the energy consumption to 0.2 megawatt-

instacourses.insightsonindia.com 21

https://upscmaterial.online/
Download From - https://upscmaterial.online/

.
Total Marks : 200.00
Test-26 (Subject)
( Insta Prelims Test Series 2023 )

hour per ton of CO2.

Other advanced technologies will not provide energy-efficient or timely solutions to


CO2 emission from conventional coal-fired power plants.

Source: https://climate.mit.edu/explainers/carbon-capture

https://www.science.org/doi/10.1126/science.1176731

21. Consider the following statements:


1. The Carbon Border Adjustment Mechanism is formulated by India to tax carbon-intensive
products such from 2026.
2. The carbon border tax involves imposing an import duty on a product manufactured in a
country with more lax climate rules than the one buying it.
3. The Carbon Border Adjustment Mechanism is widely criticised for shifting of responsibilities from
developed to developing countries.

Which of the statements given above is/are correct?


A. 1 only
B. 1 and 2 only
C. 2 and 3 only
D. 1, 2 and 3

instacourses.insightsonindia.com 22

https://upscmaterial.online/
Download From - https://upscmaterial.online/

.
Total Marks : 200.00
Test-26 (Subject)
( Insta Prelims Test Series 2023 )

Correct Answer : C

Answer Justification :

Explanation:

Statement 1 is not correct: The Carbon Border Adjustment Mechanism is a plan from the
European Union (EU) to tax carbon-intensive products, such as iron and steel, cement, fertiliser,
aluminium and electricity generation, from 2026.

Statement 2 is correct: The carbon border tax involves imposing an import duty on a product
manufactured in a country with more lax climate rules than the one buying it.

The BASIC group, comprising India, China, Brazil and South Africa, stated “unilateral measures
and discriminatory practices, such as carbon border taxes, that could result in market distortion
and aggravate the trust deficit amongst Parties, must be avoided.

Statement 3 is correct: BASIC countries call for a united solidarity response by developing
countries to any unfair shifting of responsibilities from developed to developing countries.

Source:
https://indianexpress.com/article/explained/explained-climate/carbon-border-tax-whi
ch-india-opposed-cop27-8274506/

22. With reference to Coral Bleaching, consider the following statements:


1. It happens when the corals experience high levels of ocean acidity.
2. The El Niño weather pattern acts as a contributing factor to coral bleaching.
3. Reefs once bleached cannot be revived due mainly to the weed settlement.

Which of the statements given above is/are correct?


A. 1 only
B. 1 and 2 only
C. 2 and 3 only
D. 1, 2 and 3

Correct Answer : B

Answer Justification :

Explanation:

Statement 1 is correct: Bleaching happens when corals experience stress in their


environment due to changes in temperature, pollution or high levels of ocean acidity.

Statement 2 is correct: The first mass bleaching event had occurred in 1998 when the El

instacourses.insightsonindia.com 23

https://upscmaterial.online/
Download From - https://upscmaterial.online/

.
Total Marks : 200.00
Test-26 (Subject)
( Insta Prelims Test Series 2023 )

Niño weather pattern caused sea surfaces in the Pacific Ocean to heat up; this event caused 8%
of the world’s coral to die. The second event took place in 2002.

Bleached corals can survive depending on the levels of bleaching and the recovery of sea
temperatures to normal levels.

If heat-pollutions subside in time, over a few weeks, the zooxanthellae can come back to the
corals and restart the partnership but severe bleaching and prolonged stress in the external
environment can lead to coral death

Statement 3 is not correct: Dead reefs can revive over time if there are enough fish species
that can graze off the weeds that settle on dead corals, but it takes almost a decade for the reef
to start setting up again.

The reefs which were severely damaged in 1998 did recover over time.

Source:
https://www.thehindu.com/sci-tech/energy-and-environment/great-barrier-reef-coral-
bleaching-australia-explained/article65261961.ece

23. Which of the following sub-schemes is/are included under the purview of National
Bioenergy Programme?
1. Biogas Programme
2. Renewable Energy Programme
3. Waste to Energy Programme

Select the correct answer using the code given below:


A. 1 only
B. 1 and 3 only
C. 2 and 3 only
D. 1, 2 and 3

Correct Answer : B

Answer Justification :

Explanation:

Option (b) is correct: The Ministry of New and Renewable Energy (MNRE), Government of
India has notified the National Bioenergy Programme.

MNRE has continued the National Bioenergy Programme for the period from FY 2021-22 to
2025-26. The Programme has been recommended for implementation in two Phases.

The National Bioenergy Programme will comprise of the following sub-schemes:

instacourses.insightsonindia.com 24

https://upscmaterial.online/
Download From - https://upscmaterial.online/

.
Total Marks : 200.00
Test-26 (Subject)
( Insta Prelims Test Series 2023 )

Waste to Energy Programme (Programme on Energy from Urban, Industrial and


Agricultural Wastes /Residues) to support setting up of large Biogas, Bio-CNG and Power
plants (excluding MSW to Power projects).

Biomass Programme (Scheme to Support Manufacturing of Briquettes & Pellets and


Promotion of Biomass (non-bagasse) based cogeneration in Industries) to support setting
up of pellets and briquettes for use in power generation and non-bagasse-based power
generation projects.

Biogas Programme to support setting up of family and medium size Biogas in rural areas.

Source:
https://pib.gov.in/PressReleaseIframePage.aspx?PRID=1874209&RegID=3&LID=1

24. Consider the following statements regarding the CITES (Convention on International
Trade in Endangered Species of Wild Fauna and Flora):
1. India voluntarily agreed to be bound by this convention when it entered into force.
2. All import and export species covered under CITES must be authorised through a permit
system.
3. CITES Appendix I includes species not threatened with extinction but in trade are strictly
regulated.

Which of the statements given above is/are not correct?


A. 1, 2 and 3
B. 1 and 2 only
C. 3 only
D. 1 only

Correct Answer : C

Answer Justification :

Explanation:

CITES is an international agreement between governments 184 at present, to ensure that


international trade in wild animals and plants does not threaten the survival of the species.

Statement 1 is correct: The convention entered into force in 1975 and India became the 25th
party and a state that voluntarily agrees to be bound by the Convention in 1976.

Statement 2 is correct: All import, export and re-export of species covered under CITES must
be authorised through a permit system.

instacourses.insightsonindia.com 25

https://upscmaterial.online/
Download From - https://upscmaterial.online/

.
Total Marks : 200.00
Test-26 (Subject)
( Insta Prelims Test Series 2023 )

Statement 3 is not correct: CITES Appendix I lists species threatened with extinction import
or export permits for these are issued rarely and only if the purpose is not primarily commercial.

CITES Appendix II includes species not necessarily threatened with extinction but in which trade
must be strictly regulated.

Every two years, the Conference of the Parties (CoP), the supreme decision-making body of
CITES, applies a set of biological and trade criteria to evaluate proposals from parties to decide
if a species should be in Appendix I or II.

Source:
https://indianexpress.com/article/explained/ivory-trade-india-cites-vote-nairobi-8284
057/

25. The Adaptation Gap Report 2022 has been recently released by-

A. United Nations Environment Programme


B. Wildlife Trust of India
C. International Union for Conservation of Nature
D. Global Environment Facility

Correct Answer : A

Answer Justification :

Explanation:

Option (a) is correct: The United Nations Environment Programme (UNEP) launched the 2022
edition of the Adaptation Gap Report in United Nations Climate Change Conference (COP27).

UNEP’s Adaptation Gap Report 2022: Too Little, Too Slow – Climate adaptation failure puts world
at risk finds that the world must urgently increase efforts to adapt to these impacts of climate
change.

The report looks at the benefits of prioritizing actions that both reduce greenhouse gas
emissions and help communities adapt, such as nature-based solutions, and calls for countries
to step up funding and implementation of adaptation actions.

The United Nations Environment Programme (UNEP) has been the global authority that sets the
environmental agenda, promotes the coherent implementation of the environmental dimension
of sustainable development within the UN system and serves as an authoritative advocate for
the global environment.

Source: https://www.unep.org/about-un-environment

instacourses.insightsonindia.com 26

https://upscmaterial.online/
Download From - https://upscmaterial.online/

.
Total Marks : 200.00
Test-26 (Subject)
( Insta Prelims Test Series 2023 )

26. Consider the following:


The purpose of Mission LiFE is-

A. To study a potentially habitable environment in the Jezero Crater of the Martian equator.
B. To mobilise Indians and other global citizens to take collective action for preserving the
environment.
C. To build space bricks that can be used to construct building-like structures on Mars.
D. To conserve and sustainably develop the Pantanal which is the world’s largest tropical
wetland.

Correct Answer : B

Answer Justification :

Explanation:

Option (b) is correct: The concept of LiFE was introduced by the Indian Prime Minister at
COP26 at Glasgow.

On 5 June 2022, on World Environment Day, India furthered the vision of LiFE by
launching the LiFE Global Movement, inviting academicians, researchers and start-ups
across the world to think about specific and scientific ways in which the full potential of
collective action can be harnessed to address the environment crisis.

Mission LiFE will action the ideas and ideals of LiFE through a mission-mode, scientific and
measurable programme and demonstrate India’s commitment to walk the talk on climate
change.

Mission LiFE is designed with the objective to mobilise at least one billion Indians and
other global citizens to take individual and collective action for protecting and preserving
the environment in the period 2022 to 2027.

Within India, at least 80% of all villages and urban local bodies are aimed to become
environment-friendly by 2028.

Source: https://pib.gov.in/PressReleasePage.aspx?PRID=1869550

27. Consider the following statements regarding the Kalanamak Rice:


1. This variety of rice is cultivated only in the Terai region of Uttar Pradesh.
2. This variety is prone to lodging which has badly impacted the grain filling.
3. This rice variety is protected under the Geographical Indication tag system.
instacourses.insightsonindia.com 27

https://upscmaterial.online/
Download From - https://upscmaterial.online/

.
Total Marks : 200.00
Test-26 (Subject)
( Insta Prelims Test Series 2023 )

This rice variety is protected under the Geographical Indication tag system.
A. 1 only
B. 2 only
C. 2 and 3 only
D. 1 and 3 only

Correct Answer : C

Answer Justification :

Explanation:

Statement 1 is not correct: Grown in 11 districts of the Terai region of north-eastern Uttar
Pradesh and in Nepal, the traditional variety of Kalanamak Rice has been prone to lodging, a
reason for its low yield.

Statement 2 is correct: The problem with the traditional variety of Kalanamak paddy is that
it’s tall and prone to lodging, which badly impacted grain filling and quality.

The yield, as a result, fell drastically, and the market for the rice dwindled, too.

Statement 3 is correct: The traditional Kalanamak rice is protected under the Geographical
Indication (GI) tag system.

It’s recorded in the GI application that Lord Budhha gifted Kalanamak paddy to the people of
Sravasti so that they remembered him by its fragrance.

The traditional Kalanamak paddy’s yield is barely two to 2.5 tonnes per hectare.

Lodging is a condition in which the top of the plant becomes heavy because of grain formation,
the stem becomes weak, and the plant falls on the ground.

Source:
https://www.thehindu.com/sci-tech/agriculture/fragrant-and-nutritious-kalanamak-ric
e-buddhas-gift-to-people-gets-new-powers-and-name/article66069818.ece

28. Consider the following statements:


1. The Mangrove forests grow only at tropical and subtropical latitudes near the equator.
2. The Mangrove trees can sequester up to four times more carbon than tropical rainforests.
3. The MISHTI programme facilitates the mangrove plantation along India’s coastline and on salt
pan lands.

Which of the statements given above is/are correct?


A. 1 only
B. 1 and 2 only
C. 2 and 3 only

instacourses.insightsonindia.com 28

https://upscmaterial.online/
Download From - https://upscmaterial.online/

.
Total Marks : 200.00
Test-26 (Subject)
( Insta Prelims Test Series 2023 )

D. 1, 2 and 3

Correct Answer : D

Answer Justification :

Explanation:

MISHTI (Mangrove Initiative for Shoreline Habitats & Tangible Incomes)

Statement 1 is correct: Mangroves are salt-tolerant plant communities found in tropical and
subtropical intertidal regions.

They are important refuges of coastal biodiversity and also act as bio-shields against extreme
climatic events.

Statement 2 is correct: Furthermore, they are also excellent carbon sinks. Mangrove trees
can grow in saline waters, and can sequester up to four times more carbon than tropical
rainforests.

Statement 3 is correct: MISHTI is a new programme that will facilitate mangrove plantation
along India’s coastline and on salt pan lands. The programme will operate through
“convergence between MGNREGS, Campa Fund and other sources.

Mangroves are not just some of the most bio-diverse locations in India, they also protect the
coastlines from the vagaries of inclement weather.

Restoration of the land and allowing intertidal flow is crucial for plantation and survival of
mangrove forests.

Source:
https://www.thehindu.com/sci-tech/energy-and-environment/explained-why-have-ma
ngroves-got-a-budget-push/article66472109.ece

https://indianexpress.com/article/explained/explained-climate/mishti-amrit-dharohar-
pm-pranam-governments-new-conservation-
initiatives-8417929/#:~:text=MISHTI%20%28Mangrove%20Initiative%20for%20Shore
line%20Habitats%20%26%20Tangible,Campa%20Fund%20and%20other%20sources%
2C%E2%80%9D%20the%20FM%20said.

29. The ‘SATAT Scheme’ was launched in India with the aim to-

A. To create a carbon neutral Ladakh and put India on the geothermal power map of the
world.
B. To promote solar power as a sustainable way to transition to a carbon-neutral future.
C. To target domestic manufacturing of electrolysers and production of Green Hydrogen.

instacourses.insightsonindia.com 29

https://upscmaterial.online/
Download From - https://upscmaterial.online/

.
Total Marks : 200.00
Test-26 (Subject)
( Insta Prelims Test Series 2023 )

D. To establish an ecosystem for production of Compressed Bio Gas from biomass sources in
India.

Correct Answer : D

Answer Justification :

Option (d) is correct: The Sustainable Alternative towards Affordable Transportation (SATAT)
scheme:

It was launched by Government of India to establish an ecosystem for production of


Compressed Bio Gas (CBG) from various waste/biomass sources in the country.

The scheme aims to empower the rural economy by supporting farmers, increase India's
domestic energy production and self-sufficiency and also reduce the air pollution, and
help India lead the world toward a clean energy transition.

38 CBG / Biogas Plants have been commissioned under the SATAT initiative.

Source: https://pib.gov.in/PressReleasePage.aspx?PRID=1868887

30. Consider the following statements:


1. The Blue Flag is a voluntary tag given to environment-friendly and clean beaches with hygienic
facilities.
2. The Blue Flag certification was originally implemented in India as an environmental awareness
initiative.

Which of the statements given above is/are correct?


A. 1 only
B. 2 only
C. Both 1 and 2
D. Neither 1 nor 2

Correct Answer : A

Answer Justification :

Explanation:

The Blue Flag is a voluntary tag given to environment-friendly and clean beaches with hygienic
facilities.

instacourses.insightsonindia.com 30

https://upscmaterial.online/
Download From - https://upscmaterial.online/

.
Total Marks : 200.00
Test-26 (Subject)
( Insta Prelims Test Series 2023 )

Statement 1 is correct: The tag is part of an eco-tourism model that promotes sustainability
in the tourism sector through environmental awareness, protection and sustainable
development practices.

It is accorded by Denmark-based non-profit Foundation for Environmental Education, or FEE, as


part of its Blue Flag Programme.

The Blue Flag Programme started in France in 1985 and has been implemented in Europe since
1987, and in areas outside Europe since 2001, when South Africa joined.

Statement 2 is not correct: In France, the programme originally started as an environmental


awareness initiative.

Broadly, they are divided into four categories, covering water quality, environmental
management, safety, and environmental information and education.

Source:
https://www.thehindu.com/news/national/blue-flag-beach-award-lakshadweep-certific
ate-explain/article66064624.ece

https://pib.gov.in/PressReleasePage.aspx?PRID=1871022

31. With reference to Glyphosate, consider the following statements:


1. It is a synthetic organophosphate compound used as a broad-spectrum pesticide.
2. In India it is prescribed for use in tea plantations as well as in paddy fields before sowing.

Which of the statements given above is/are correct?


A. 1 only
B. 2 only
C. Both 1 and 2
D. Neither 1 nor 2

Correct Answer : C

Answer Justification :

Explanation:

Glyphosate is the active ingredient in weed killer products.

Glyphosate products are one of the most widely used weed killers worldwide in farms and in
home gardens and lawns.

Statement 1 is correct: Glyphosate is a synthetic organophosphate compound that blocks the


activity of enolpyruvylshikimate-3-phosphate (EPSP) synthase and is used as a broad-spectrum
pesticide.

instacourses.insightsonindia.com 31

https://upscmaterial.online/
Download From - https://upscmaterial.online/

.
Total Marks : 200.00
Test-26 (Subject)
( Insta Prelims Test Series 2023 )

It is characterized as a volatile, moderately toxic, odourless crystalline solid or powder, and


exposure occurs by inhalation, ingestion, or contact.

Statement 2 is correct: Although prescribed for use in tea plantations and fallow land,
Glyphosate’s is now widely used in paddy fields before sowing.

Nowadays, it is also used for crops like pineapple and banana and even in households for
removing grass from between interlocking tiles.

Source:
https://www.thehindu.com/news/national/kerala/weedicide-glyphosate-banned/article
26176536.ece

https://www.fda.gov/food/pesticides/questions-and-answers-glyphosate

32. Consider the following statements regarding the Dark Sky Reserves:
1. It is a designation given to a place that has policies in place to ensure that a tract of land has
minimal artificial light interference.
2. The International Dark Sky Association is an inter-governmental organisation that designates
places as International Dark Sky Places.

Which of the statements given above is/are not correct?


A. 1 only
B. 2 only
C. Both 1 and 2
D. Neither 1 nor 2

Correct Answer : B

Answer Justification :

Explanation:

Statement 1 is correct: Dark Sky Reserve is a designation given to a place that has policies in
place to ensure that a tract of land or region has minimal artificial light interference.

Individuals or groups can nominate a site for certification to the International Dark Sky
Association (IDSA).

There are five designated categories, namely International Dark Sky parks, communities,
reserves, sanctuaries and Urban Night Sky Places.

Statement 2 is not correct: The International Dark Sky Association is a U.S.-based non-profit
organisation that designates places as International Dark Sky Places, Parks, Sanctuaries and
Reserves, depending on the criteria they meet.

instacourses.insightsonindia.com 32

https://upscmaterial.online/
Download From - https://upscmaterial.online/

.
Total Marks : 200.00
Test-26 (Subject)
( Insta Prelims Test Series 2023 )

Between 2001 and January 2022, there have been 195 sites recognised as International Dark
Sky Places globally.

Source:
https://indianexpress.com/article/explained/explained-sci-tech/ladakh-dark-sky-reser
ve-to-promote-astronomy-tourism-8132171/

https://www.thehindu.com/sci-tech/science/the-dark-sky-is-a-natural-resource-and-to
o-much-light-is-polluting-it/article66423147.ece

33. With reference to Ocean Acidification, consider the following statements:


1. It refers to the reduction in pH of the ocean primarily due to the uptake of carbon dioxide from
the atmosphere.
2. Oysters and clams cannot make or maintain their shells because of the acidic ocean water.
3. Acidic-ocean water can weaken the reefs but create optimum conditions for the corals to
multiply at a rapid speed.

Which of the statements given above is/are correct?


A. 1 only
B. 1 and 2 only
C. 1 and 2 only
D. 1, 2 and 3

Correct Answer : B

Answer Justification :

Explanation:

Ocean acidification is a change in the properties of ocean water that can be harmful for plants
and animals.

Statement 1 is correct: Scientists have observed that the ocean is becoming more acidic as
its water absorbs carbon dioxide from the atmosphere.

Just like acids can cause rust and other damage to surfaces on land, acidic ocean water can
affect surfaces underwater.

Statement 2 is correct: Acids can break down the shells of animals that live in the sea.
Because ocean water has become more acidic, some animals like certain oysters and clams are
having difficulty in making or keeping their shells.

Statement 3 is not correct: For example, acidic ocean water can cause coral to grow more
slowly and weaken coral reefs. These reefs are an important home for many living things. Their
health is essential to many ecosystems.

instacourses.insightsonindia.com 33

https://upscmaterial.online/
Download From - https://upscmaterial.online/

.
Total Marks : 200.00
Test-26 (Subject)
( Insta Prelims Test Series 2023 )

Source: https://climatekids.nasa.gov/acid-ocean/

https://oceanservice.noaa.gov/facts/acidification.html

34. Which one of the following statements is correct about the Rotterdam Convention?

A. It is a multilateral treaty to promote shared responsibilities in relation to importation of


hazardous chemicals.
B. It is an international treaty that seeks to protect human health and the environment from
anthropogenic emissions.
C. It applies to all activities involving exposure of workers to asbestos in the course of work.
D. It regulates transboundary movements of hazardous wastes and ensures that wastes
are disposed in environmentally sound manner.

Correct Answer : A

Answer Justification :

Explanation:

Option (a) is correct: The Rotterdam Convention (formally, the Rotterdam Convention on the
Prior Informed Consent Procedure for Certain Hazardous Chemicals and Pesticides in
International Trade) is a multilateral treaty to promote shared responsibilities in relation to
importation of hazardous chemicals.

The convention promotes open exchange of information and calls on exporters of hazardous
chemicals to use proper labelling, include directions on safe handling, and inform purchasers of
any known restrictions or bans.

Option (b) is not correct: The Minamata Convention on Mercury is an international treaty
that seeks to protect human health and the environment from anthropogenic emissions and
releases of mercury and mercury compounds.

Option (c) is not correct: Asbestos Convention applies to all activities involving exposure of
workers to asbestos in the course of work.

Option (d) is not correct: The Basel Convention on the Control of Transboundary Movements
of Hazardous Wastes and their Disposal was adopted in 1989 and it came into force in 1992.

It is the most comprehensive global environmental agreement on hazardous wastes and other
wastes.

Source:
https://www.unep.org/resources/report/basel-convention-control-transboundary-mov
ements-hazardous-wastes

instacourses.insightsonindia.com 34

https://upscmaterial.online/
Download From - https://upscmaterial.online/

.
Total Marks : 200.00
Test-26 (Subject)
( Insta Prelims Test Series 2023 )

https://www.ilo.org/dyn/normlex/en/f?p=1000:12100:::NO:12100:P12100_INSTRUMEN
T_ID:312307

https://chemicals.gov.in/rotterdam-convention

35. With reference to the Coal Gasification, consider the following statements:
1. It is a technological process that can convert any carbonaceous raw material into fuel gas.
2. Syngas produced in the process is a derivative of hydrogen and does not release carbon
monoxide or carbon dioxide.
3. India’s coal gasification project is aimed at meeting its commitments under CoP-21 Paris
Agreement.

Which of the statements given above is/are correct?


A. 1 only
B. 2 only
C. 1, 2 and 3
D. 1 and 3 only

Correct Answer : D

Answer Justification :

Explanation:

Statement 1 is correct: Coal gasification is a process that converts carbonaceous materials,


such as coal, petroleum, petroleum coke, or biomass, into carbon monoxide and hydrogen.

Statement 2 is not correct: Coal gasification is the process of converting coal into synthesis
gas (also called syngas), which is a mixture of hydrogen (H2), carbon monoxide (CO) and
carbon dioxide (CO2).

The syngas can be used in a variety of applications such as in the production of electricity and
making chemical products, such as fertilisers.

The coal gasification process holds good potential in the future, with coal being the most
abundantly available fossil fuel across the world, and that even low-grade coal can be used in
the process.

Statement 3 is correct: Recently Odisha’s Talcher fertiliser plant was awarded a contract for
starting a coal gasification unit for the production of urea and Ammonia.

It was part of the government’s initiative to revive closed fertiliser plants belonging to the
Fertiliser Corporation of India Limited (FCIL) and the Hindustan Fertilisers Corporation Ltd
(HFCL).

The project’s environment-friendliness would help India in meeting its commitments under the

instacourses.insightsonindia.com 35

https://upscmaterial.online/
Download From - https://upscmaterial.online/

.
Total Marks : 200.00
Test-26 (Subject)
( Insta Prelims Test Series 2023 )

CoP-21 Paris Agreement.

Source:
https://indianexpress.com/article/explained/explained-how-coal-gasification-can-help-
india-bring-down-its-urea-import-6025114/

https://www.netl.doe.gov/research/Coal/energy-systems/gasification/gasifipedia/intro
-to-gasification

36. Which of the following is/are the advantages of Direct Seeding?


1. Zero Greenhouse Gas emissions
2. Zero plant stress from transplanting
3. Faster maturation of crops
4. Increases income by reducing cost of cultivation

Select the correct answer using the code given below:


A. 1 and 2 only
B. 3 and 4 only
C. 1 and 3 only
D. 2, 3 and 4 only

Correct Answer : D

Answer Justification :

Explanation:

Option (d) is correct: Direct seeded crops require less labour and tend to mature faster than
transplanted crops.

In this method, plants are not subjected to stresses such as being pulled from the soil and re-
establishing fine rootlets. However, they have more competition from weeds.

Advantages of direct seeding

No significant reduction of yield under optimal conditions.

Savings on irrigation water by 12-35% under efficient water management practices.

Reduces labour and drudgery by eliminating seedling uprooting and transplanting.

Reduces cultivation time, energy, and cost.

instacourses.insightsonindia.com 36

https://upscmaterial.online/
Download From - https://upscmaterial.online/

.
Total Marks : 200.00
Test-26 (Subject)
( Insta Prelims Test Series 2023 )

No plant stress from transplanting.

Faster maturation of crops.

Lower GHG emissions.

Increases total income by reducing cost of cultivation.

Source: https://dsrc.irri.org/our-work/what-is-dsr

http://www.knowledgebank.irri.org/step-by-step-production/growth/planting/direct-s
eeding

37. Consider the following statements:


1. The Global Solar Council was established during COP21 to promote the rapid adoption of solar
energy globally.
2. The Global Solar Council is a member of the Marrakech Partnership for Global Climate Action
within the United Nations Framework on Climate Change.

Which of the statements given above is/are correct?


A. 1 only
B. 2 only
C. Both 1 and 2
D. Neither 1 nor 2

Correct Answer : C

Answer Justification :

Explanation:

Statement 1 is correct: The Global Solar Council (GSC) has been established in during COP21
in Paris in 2015, with the objective to promote the rapid adoption of solar energy globally,
through market development, partnerships and education.

Statement 2 is correct: Since then, the GSC has grown a strong presence in the Climate
Policy space, and in 2020 became a member of the Marrakech Partnership for Global Climate
Action (MPGCA) within the United Nations Framework on Climate Change (UNFCCC).

GSC aims to bring clear policy statements to Government stakeholders and maximize
engagement and communication opportunities to show the potential of Solar PV and industry
members.

instacourses.insightsonindia.com 37

https://upscmaterial.online/
Download From - https://upscmaterial.online/

.
Total Marks : 200.00
Test-26 (Subject)
( Insta Prelims Test Series 2023 )

The Marrakech Partnership for Global Climate Action is an active process and platform with the
objective to enable collaboration between governments and cities, regions, businesses, and
investors to accelerate climate action.

Source: https://www.globalsolarcouncil.org/pages/gsc-at-cop27/

38. Which one of the following statements is correct about the Pala Wetland?

A. This wetland is the largest natural wetland in the state of Mizoram.


B. This wetland contains an entire National Park in its low-lying area.
C. The marshy areas in the wetland provide excellent habitat for sambar deer.
D. The wetland is revered by the Mara people and is a source for irrigation.

Correct Answer : B

Answer Justification :

Explanation:

Option (a) is correct: Pala Wetland is the largest natural wetland in the state of Mizoram. The
Site includes a relatively deep lake and its surrounding forest catchment area which climbs to
almost 600m above sea level.

Option (b) is not correct: Loktak Lake of Manipur is famous for its Phumdis. The largest
recorded Phumdis contain an entire National Park known as Keibul Lamjao National Park.

Option (c) is correct: The low-lying marshy areas within the wetland provide excellent habitat
for many herbivores such as sambar deer (Cervus unicolor), wild pig (Sus scrofa) and barking
deer (Muntiacus muntjak).

Various species of primate also inhabit the wetland, such as the endangered Hoolock gibbon
(Hoolock hoolock) and Phayre’s leaf monkey (Trachypithecus phayrei).

Option (d) is correct: Pala Wetland is revered by the local Mara people and has a deep
connection with their history.

The wetland is the major source of freshwater and fish for the villages at its fringe and also the
major source of irrigation water for wet rice cultivation, horticulture and other forms of
agriculture.

Source: https://www.worldatlas.com/lakes/loktak-lake.html

https://rsis.ramsar.org/ris/2484

instacourses.insightsonindia.com 38

https://upscmaterial.online/
Download From - https://upscmaterial.online/

.
Total Marks : 200.00
Test-26 (Subject)
( Insta Prelims Test Series 2023 )

39. Consider the following statements:


1. India has joined the First Movers Coalition which is aimed at decarbonising the heavy industry
responsible for 30 per cent of global emissions.
2. The First Movers Coalition is led by United Nations Framework Convention on Climate Change.

Which of the statements given above is/are correct?


A. 1 only
B. 2 only
C. Both 1 and 2
D. Neither 1 nor 2

Correct Answer : A

Answer Justification :

Explanation:

Statement 1 is correct: India has joined the First Movers Coalition, a global initiative aimed at
decarbonising the heavy industry and long-distance transport sectors responsible for 30 per
cent of global emissions.

At the 2021 United Nations Climate Change Conference (COP26), the United States announced
the launch of the First Movers Coalition, a new platform for companies to harness their
purchasing power and supply chains to create early markets for innovative clean energy
technologies that are key for tackling the climate crisis.

Statement 2 is not correct: Led by the World Economic Forum and the US government, the
First Movers Coalition targets sectors, including aluminium, aviation, chemicals, concrete,
shipping, steel and trucking.

India, along with Japan and Sweden, has also joined the steering board of the coalition.

Source:
https://www.business-standard.com/article/economy-policy/india-joins-first-movers-c
oalition-to-decarbonise-carbon-heavy-sectors-122052500887_1.html

https://www.state.gov/launching-the-first-movers-coalition-at-the-2021-un-climate-ch
ange-conference/

40. Consider the following statements regarding the Great Indian Bustards (GIBs):
1. GIBs are considered as the barometers of health of grassland ecosystems.
2. One of the major threats to the GIBs are overhead power transmission lines.
3. GIBs are declared critically endangered by the International Union for Conservation of Nature.

Which of the statements given above is/are correct?

instacourses.insightsonindia.com 39

https://upscmaterial.online/
Download From - https://upscmaterial.online/

.
Total Marks : 200.00
Test-26 (Subject)
( Insta Prelims Test Series 2023 )

A. 1 and 2 only
B. 2 and 3 only
C. 1 and 3 only
D. 1, 2 and 3

Correct Answer : D

Answer Justification :

Explanation:

Statement 1 is correct: GIBs are considered the flagship bird species of grassland and hence
barometers of the health of grassland ecosystems.

Statement 2 is correct: Among the biggest threats to the GIBs are overhead power
transmission lines. Due to their poor frontal vision, the birds can’t spot the power lines from a
distance, and are too heavy to change course when close. Thus, they collide with the cables and
die.

Statement 3 is correct: Listed in Schedule I of the Indian Wildlife (Protection) Act, 1972, in
Appendix I of CITES, as Critically Endangered on the IUCN Red List, the GIBs enjoy the highest
protection both in India and globally.

According to scientists, the GIBs are slow breeders and they build their nests on the ground.
The species have also been subjected to hunting and egg collection in the past.

There also has been a decline in prevailing habitat loss as dry grasslands have been diverted for
other use.

Source:
https://www.thehindu.com/sci-tech/energy-and-environment/explained-protecting-the-great-indi
an-bustard/article66219920.ece

https://indianexpress.com/article/explained/supreme-court-great-indian-bustard-project-explain
ed-8299659/

41. The Cloud Forest Assets Programme focuses on-

A. A weather modification technique that improves a cloud’s ability to produce rain by


artificially adding condensation nuclei to the atmosphere.
B. A specific tropical forest ecosystem that offers the potential to be financed as green
infrastructure.
C. A radar network over the cloud burst-prone areas with high-resolution weather
forecasting models.
D. Regeneration and eco-development of degraded forests and adjoining areas on a

instacourses.insightsonindia.com 40

https://upscmaterial.online/
Download From - https://upscmaterial.online/

.
Total Marks : 200.00
Test-26 (Subject)
( Insta Prelims Test Series 2023 )

watershed basis.

Correct Answer : B

Answer Justification :

Explanation:

Option (b) is correct: The Cloud Forest Assets programme focuses on a specific tropical forest
ecosystem that provides a crucial service of water stability, offering the potential to be financed
as green infrastructure.

The programme works with stakeholders to design innovative financial mechanisms to enable
governments, investors, companies and communities to monetise the hydrological function of
forests, in addition to the carbon they store, and develop financial opportunities that reward
nature protection.

Cloud forests are mountain tropical forests, constantly shrouded in mist, which sit at the
headwater of river basins.

Source: https://www.earthsecurity.org/what-we-do

https://moef.gov.in/wp-content/uploads/2017/09/OPERATIONALGUIDELINES.pdf

42. With reference to Ganges River dolphin, consider the following statements:
1. They can only live in freshwater and are essentially blind.
2. They cannot breathe in water and come to surface in short durations.
3. The movement of these dolphins is independent of seasonal patterns.

Which of the statements given above is/are correct?


A. 1 only
B. 3 only
C. 2 and 3 only
D. 1 and 2 only

Correct Answer : D

Answer Justification :

Explanation:

Statement 1 is correct: The Ganges River dolphins can only live in freshwater and are
essentially blind.

They hunt by emitting ultrasonic sounds waves that bounce off of fish and other prey.

instacourses.insightsonindia.com 41

https://upscmaterial.online/
Download From - https://upscmaterial.online/

.
Total Marks : 200.00
Test-26 (Subject)
( Insta Prelims Test Series 2023 )

They are frequently found alone or in small groups; generally, a mother and calf travel together.

The dolphin has the peculiarity of swimming on one side so that its flipper trails the muddy
bottom.

Statement 2 is correct: It is understood that this behaviour aids them in finding food. Being a
mammal, the Ganges River dolphin cannot breathe in water and must surface every 30-120
seconds.

Because of the sound it produces when breathing, the animal is popularly referred to as 'Susu'.

Statement 3 is not correct: Although not well studied, the movement of the Ganges River
dolphin follows seasonal patterns. It is observed that animals travel upstream when the water
level rises, and from there enter smaller streams.

Source: https://wii.gov.in/nmcg/priority-species/mammals/ganges-river-dolphin

43. ‘SWAS, SAFAL and STAR’ recently talked about in news are associated with-

A. Weather Forecasting Initiatives


B. Air Quality Information Service
C. Categories of Green Crackers
D. Food Safety Management Standards

Correct Answer : C

Answer Justification :

Explanation:

Option (c) is correct: Green Crackers reduce emissions substantially and absorb dust and
don’t contain hazardous elements like barium nitrate.

Toxic metals in traditional crackers are replaced with less hazardous compounds.

According to the National Green Tribunal (NGT), green crackers are permitted only in
cities and towns where air quality is moderate or poor

Green crackers fall only in these three categories- SWAS, SAFAL and STAR: the crackers
developed by the Council of Scientific and Industrial Research (CSIR).

“SWAS, is “safe water releaser” should have a small-water pocket/droplets which get
released when burst, in the form of vapour.

instacourses.insightsonindia.com 42

https://upscmaterial.online/
Download From - https://upscmaterial.online/

.
Total Marks : 200.00
Test-26 (Subject)
( Insta Prelims Test Series 2023 )

“SWAS is safe water releaser, which suppresses the dust released by releasing water
vapour in the air.

It does not comprise potassium nitrate and sulphur and the particulate dust released will
reduce approximately by 30 per cent.

Similarly, STAR is the safe thermite cracker, which does not comprise potassium nitrate
and sulphur, emits reduced particulate matter disposal and reduced sound intensity.

SAFAL is safe minimal aluminium which has minimum usage of aluminium, and used
magnesium instead. It ensures reduction in sound in comparison to traditional crackers.

Source:
https://indianexpress.com/article/explained/green-crackers-identify-them-8214961/

44. With reference to Eutrophication, consider the following statements:


1. It triggers algal growth on the surface of the water body which prevents penetration of sunlight.
2. It inhibits photosynthetic activity in the water body and creates hypoxic for the aquatic
community.

Which of the statements given above is/are correct?


A. 1 only
B. 2 only
C. Both 1 and 2
D. Neither 1 nor 2

Correct Answer : C

Answer Justification :

Explanation:

Eutrophication occurs when the environment becomes enriched with nutrients, increasing the
amount of plant and algae growth to estuaries and coastal waters.

Statement 1 is correct: Major threat to freshwater ecology is eutrophication i.e the excessive
build-up of nutrients.

Eutrophication triggers algal growth on the surface of the water body, which prevents
penetration of sunlight.

Statement 2 is correct: This inhibits photosynthetic activity in the water body and creates

instacourses.insightsonindia.com 43

https://upscmaterial.online/
Download From - https://upscmaterial.online/

.
Total Marks : 200.00
Test-26 (Subject)
( Insta Prelims Test Series 2023 )

hypoxic (depleted in oxygen conditions) for the aquatic community.

Source: https://oceanservice.noaa.gov/facts/eutrophication.html

https://indianexpress.com/article/technology/science/pesticide-bacterioplankton-poll
ution-battle-7463097/

45. Consider the following passage:


1. It is a multilateral environmental agreement that regulates the production and consumption of
man-made chemicals referred to as ozone depleting substances (ODS). It phases down the
consumption and production of the different ozone depleting substances in a step-wise manner.

Which one of the following protocols is best described in the passage given above?
A. Montreal Protocol
B. Kyoto Protocol
C. Nagoya Protocol
D. Cartagena Protocol

Correct Answer : A

Answer Justification :

Explanation:

Option (a) is correct: The Montreal Protocol on Substances that Deplete the Ozone Layer is
the landmark multilateral environmental agreement that regulates the production and
consumption of nearly 100 man-made chemicals referred to as ozone depleting substances
(ODS).

When released to the atmosphere, those chemicals damage the stratospheric ozone layer,
Earth’s protective shield that protects humans and the environment from harmful levels of
ultraviolet radiation from the sun.

The Montreal Protocol phases down the consumption and production of the different ODS in a
step-wise manner, with different timetables for developed and developing countries.

Source: https://www.unep.org/ozonaction/who-we-are/about-montreal-protocol

46. Consider the following statements regarding the Environmental Protection Agency
(EPA):
1. It is an extension of the United Nations Environment Programme to protect environmental
health.
2. Major areas covered by the EPA include wildlife, food safety and nuclear waste.
3. It regulates the manufacturing, processing and distribution of chemicals and other pollutants.

instacourses.insightsonindia.com 44

https://upscmaterial.online/
Download From - https://upscmaterial.online/

.
Total Marks : 200.00
Test-26 (Subject)
( Insta Prelims Test Series 2023 )

Which of the statements given above is/are not correct?


A. 1 only
B. 3 only
C. 1 and 2 only
D. 2 and 3 only

Correct Answer : C

Answer Justification :

Explanation:

Statement 1 is not correct: The Environmental Protection Agency is a United States federal
government agency whose mission is to protect human and environmental health.

Statement 2 is not correct: Some of the areas that aren’t covered by the EPA include
wildlife, wetlands, food safety, and nuclear waste.

The agency enforces its findings through fines, sanctions, and other procedures.

It oversees programs to promote energy efficiency, environmental stewardship, sustainable


growth, air and water quality, and pollution prevention.

Statement 3 is correct: The EPA regulates the manufacturing, processing, distribution, and
use of chemicals and other pollutants.

Also, the EPA is charged with determining safe tolerance levels for chemicals and other
pollutants in food, animal feed, and water.

Source:
https://www.investopedia.com/terms/e/environmental-protection-agency.asp#:~:text
=The%20Environmental%20Protection%20Agency%20is%20a%20United%20States,it
s%20findings%20through%20fines%2C%20sanctions%2C%20and%20other%20proced
ures.

Environmental Protection Agency | GlobalChange.gov

47. With reference to the Wild Life (Protection) Act of 1972, consider the following
statements:
1. It provides for establishment of sanctuaries and national parks.
2. It provides for state wildlife advisory boards.
3. It provides for judicially imposed penalties for violating the Act.

Which of the statements given above is/are correct?


A. 1 only
B. 1 and 2 only

instacourses.insightsonindia.com 45

https://upscmaterial.online/
Download From - https://upscmaterial.online/

.
Total Marks : 200.00
Test-26 (Subject)
( Insta Prelims Test Series 2023 )

C. 2 and 3 only
D. 1, 2 and 3

Correct Answer : D

Answer Justification :

Explanation:

Option (d) is correct: The Wild Life (Protection) Act of 1972 and Amendment, 1982:

• The wild life protection Act, provides for state wildlife advisory boards, regulations for hunting
wild animals and birds, establishment of sanctuaries and national parks, regulations for trade in
wild animals, animal products and trophies, and judicially imposed penalties for violating the
Act.

• Harming endangered species listed in Schedule 1 of the Act is prohibited throughout India.
Hunting species, like those requiring special protection (Schedule II), big game (Schedule III),
and small game (Schedule IV), is regulated through licensing.

• A few species classified as vermin (Schedule V), may be hunted without restrictions. Wildlife
wardens and their staff administer the act.

• An amendment to the Act in 1982, introduced a provision permitting the capture and
transportation of wild animals for the scientific management of animal population.

Source:https://www.nios.ac.in/media/documents/333courseE/LG/eng/EVS_333_E_LG_23.pdf

48. Consider the following statements regarding the National Green Tribunal (NGT):
1. It is bound to follow procedures laid down in the Code of Civil Procedure.
2. Its order or award is executable by itself as a decree of the Civil Court.
3. The decision of Tribunal by majority is non-binding on the defaulter.

Which of the statements given above is/are correct?


A. 1 only
B. 1 and 2 only
C. 2 and 3 only
D. 1, 2 and 3

Correct Answer : B

Answer Justification :

Explanation:

instacourses.insightsonindia.com 46

https://upscmaterial.online/
Download From - https://upscmaterial.online/

.
Total Marks : 200.00
Test-26 (Subject)
( Insta Prelims Test Series 2023 )

Statement 1 is correct: NGT is not bound to follow the procedures laid down in the Code of
Civil Procedure, 1908; or the Indian Evidence Act, 1872; rather it has to be guided by the
principles of natural justice.

It has for the purposes of discharge of its functions all the powers of the Civil Court for trying a
suit as given under the Code of Civil Procedure, 1908.

Statement 2 is correct: Any decision, order or ‘award’ of the Tribunal is executable by the
Tribunal as a ‘decree’ of the Civil Court and, therefore, for this purpose the ‘Tribunal’ will have
all the powers of a Civil Court.

The Tribunal can also if it deems fit transmit its order or award for execution to a Civil Court
having local jurisdiction as if it were the ‘decree’ of that Civil Court.

Minimum number of members who must together hear and decide a case is two, out of which
one must be a judicial member and other an expert member.

Statement 3 is not correct: The decision of Tribunal by majority is binding on the defaulter.

Source: Lesson-24.pmd (nios.ac.in)

49. Consider the following statements regarding Single Use Plastics


1. More than 90% of single use plastic used globally are sourced from fossil fuels.
2. Plastics remain in the environment as microplastics, that tend to bioaccumulate in human body.
3. They can lead to greenhouse gas emissions.

Which of the statements given above is/are correct?


A. 1 and 3 only
B. 1 and 2 only
C. 3 only
D. 1, 2 and 3

Correct Answer : D

Answer Justification :

Single use plastic refers to plastic items that are used once and are then discorded

it includes plastics used in packaging items, bottles, polythene bags, face masks, coffee
cups, trash bags, food packaging and so on.

According to a 2021 report of the Minderoo foundation (an Australian philanthropic


organisation), single use plastics account for a third of all plastic produced
globally.

instacourses.insightsonindia.com 47

https://upscmaterial.online/
Download From - https://upscmaterial.online/

.
Total Marks : 200.00
Test-26 (Subject)
( Insta Prelims Test Series 2023 )

98% of all such plastics are manufactured from fossil fuels.

Hence statement 1 is correct.

It is difficult to collect and recycle single use plastic items.

Plastic items remain in the environment for a long period of time

They do not decay and turn into microplastics

Microplastics first enter our food sources and then into human body, and this is
extremely harmful for humans.

Hence statement 2 is correct.

On the current trajectory of production, it has been projected that single use plastics
could account for 5 to 10% of greenhouse gas emissions by 2050.

Hence statement 3 is correct.

50. Consider the following statements regarding Ecological Succession


1. Deforestation can transform a particular seral stage of succession to an earlier stage.
2. Phytoplankton act as pioneers in case of primary succession in water.
3. Hydrarch succession, that takes place in wetter areas leads to creation of mesic conditions.

Which of the statements given above is/are correct?


A. 1 and 3 only
B. 2 only
C. 2 and 3 only
D. 1, 2 and 3

Correct Answer : D

Answer Justification :

The entire sequence of community that successively change in a given area are called
sere(s).

instacourses.insightsonindia.com 48

https://upscmaterial.online/
Download From - https://upscmaterial.online/

.
Total Marks : 200.00
Test-26 (Subject)
( Insta Prelims Test Series 2023 )

The individual transition and communication are called seral stages or seral communities.

At any time during primary or secondary succession, natural or human induced


disturbance like (fire, deforestation, etc.,) can convert a particular seral stage
of succession to an earlier stage

Also, such disturbances create new conditions that encourage some species and
discourage or eliminate other species.

Hence statement 1 is correct.

The species that invade a bare area are called Pioneer species.

In primary succession in water the pioneers are small phytoplankton.

With time, they are replaced with free floating angiosperms and then by rooted
hydrophytes, sedges, grasses and finally the trees

The climax community in this case would be a forest

With time, the water body is converted into land.

Hence statement 2 is correct.

Based on the nature of the habitat- whether it is very wet or dry, succession of plants is
called Hydrarch or Xerarch respectively.

Hydrarch succession takes place in wetter areas and the successional series
progress from hydric to the mesic conditions.

Hence statement 3 is correct.

51. Consider the following statements regarding Biodiversity Hotspots


1. United Nations Environment Program designates the Earth’s most endangered terrestrial
ecoregions' as ‘Biodiversity Hotspots’ under the Man and Biosphere Programme.
2. A Biodiversity hotspot should contain at least 1500 species of vascular plants as endemic
species.

instacourses.insightsonindia.com 49

https://upscmaterial.online/
Download From - https://upscmaterial.online/

.
Total Marks : 200.00
Test-26 (Subject)
( Insta Prelims Test Series 2023 )

3. To be designated as a biodiversity hotspot, a region should have lost at least 70% of its original
natural vegetation.

Which of the statements given above is/are correct?


A. 2 and 3 only
B. 2 only
C. 1 and 2 only
D. 1, 2 and 3

Correct Answer : A

Answer Justification :

Biodiversity Hotspots are regions with high species richness and high degree of
endemism.

British biologist Norman Myers first provided the concept of biodiversity hotspot; he
coined the term biodiversity hotspot as a biogeographic region characterized both by
exceptional levels of plant endemism and serious levels of habitat loss.

At present, a region is designated as biodiversity hotspot by Conservation


International, which is a non-profit environmental organisation.

Hence statement 1 is incorrect.

According to Conservation International, to qualify as a hotspot,

A region must contain at least 1500 species of vascular plants (> 0.5 % of the world's
total) as endemics, that is, it must have a high percentage of plant life found nowhere
else on earth.

It should have lost at least 70% of its original habitat (i.e it must have 30% or less of its
original natural vegetation).

The region must be threatened.

Hence statements 2 and 3 are correct.

52. Consider the following pairs


Convention : Related to

instacourses.insightsonindia.com 50

https://upscmaterial.online/
Download From - https://upscmaterial.online/

.
Total Marks : 200.00
Test-26 (Subject)
( Insta Prelims Test Series 2023 )

1. Cartagena Protocol : Safe handling of living modified organisms


2. Bishkek Declaration : Conservation of cheetah
3. Nagoya Protocol : Preservation of ozone layer

How many of the pairs given above are correct?


A. None of the above
B. Only one pair
C. Only two pairs
D. All three pairs

Correct Answer : B

Answer Justification :

Cartagena Protocol

The Cartagena Protocol on Biosafety to the Convention on Biological Diversity is an


international agreement that aims to ensure safe handling, transport and use of living
modified organisms resulting from modern biotechnology that may have adverse effects
on biological diversity

It also takes into account, risks to human health

It was adopted on January 29, 2000 and came into force on September 11, 2003.

Hence pair 1 is correct.

Bishkek Declaration

It was adopted at Bishkek, Kyrgyz Republic in 2013

It was adopted by leaders in the governments of all 12 snow leopard range countries
regarding conservation of snow leopard.

Hence pair 2 is incorrect

Nagoya Protocol

It deals with Access to Genetic Resources and the fair and equitable sharing of benefits
arising from their utilisation.

instacourses.insightsonindia.com 51

https://upscmaterial.online/
Download From - https://upscmaterial.online/

.
Total Marks : 200.00
Test-26 (Subject)
( Insta Prelims Test Series 2023 )

It was signed in 2010 and aims to give effect to fair and equitable sharing provisions in
the Convention of Biological Diversity.

Hence pair 3 is incorrect.

53. Consider the following statements


1. Black carbon is emitted through incomplete combustion of fossil fuels
2. Emission of brown carbon due to burning of fossil fuel releases Tarballs.
3. Mangroves and salt marshes are referred to as blue carbon ecosystems.
4. Due to their heat and light absorption properties, both Brown and black carbon impact the
greenhouse balance of the Earth.

Which of the statements given above is/are correct?


A. 2, 3 and 4 only
B. 1 and 3 only
C. 1, 2 and 3 only
D. All of the above

Correct Answer : D

Answer Justification :

Chemically, Black carbon is a component of the fine particulate matter.

Black carbon consists of pure carbon in several linked forms.

It is formed through incomplete combustion of fossil fuels, biofuel and Biomass and is one
of the main types of particles in both anthropogenic and naturally occurring soot.

It causes human morbidity and premature mortality.

Hence statement 1 is correct.

Brown carbon is the brown smoke released due to combustion of organic


matter.

It coexists with black carbon in the atmosphere.

Types of Brown carbon include tar materials from smouldering fires or coal combustion,

instacourses.insightsonindia.com 52

https://upscmaterial.online/
Download From - https://upscmaterial.online/

.
Total Marks : 200.00
Test-26 (Subject)
( Insta Prelims Test Series 2023 )

breakdown products from Biomass burning, a mixture of organic compounds emitted from
soil and volatile organic compounds given off by vegetation.

Tarballs are small light absorbing carbonacious particles formed due to burning
of biomass or fossil fuels that deposit on snow or ice. They are formed from
brown carbon emitted during the burning of fossil fuels.

Hence statement 2 is correct.

Blue carbon refers to the carbon stored, sequestered and cycled through
coastal and ocean ecosystem like mangroves, tidal salt marshes and seagrass
meadows.

These coastal vegetated landscapes are highly productive at sequestering carbon, storing
it in above and below the surface Biomass such as stems, leaves, root material and
sediments.

Although these habitats covered less than 0.5 % of the ocean’s sea bed and shore areas,
they are responsible for 50 to 70% of all carbon absorbed by ocean sediments.

Salt marshes, for example have a carbon sequestration rate that is over 50 times faster
than Tropical rainforest

Blue carbon potential is the relevant means for controlling atmospheric Greenhouse gas
concentrations.

Hence statement 3 is correct.

Black and brown forms of carbon are both very important for climate change since their
respective light and heat absorptive properties impact the greenhouse balance of the
earth.

Up to about 70% of light absorption is by black carbon; when brown carbon is present
independently it has nearly 15% potential to warm the atmosphere by absorbing light.

These forms of carbon also adversely impact human health due to their contribution to
smog.

Hence statement 4 is correct.

instacourses.insightsonindia.com 53

https://upscmaterial.online/
Download From - https://upscmaterial.online/

.
Total Marks : 200.00
Test-26 (Subject)
( Insta Prelims Test Series 2023 )

54. Which of the following is/are primary pollutants in the atmosphere?


1. Carbon dioxide
2. Sulphur trioxide
3. Nitrogen dioxide
4. Sulphur dioxide
5. Carbon monoxide
6. Ozone

Select the correct answer using the codes given below


A. 1, 3, 4 and 5 only
B. 2, 4, 5 and 6 only
C. 1, 2, 3 and 5 only
D. All of the above

Correct Answer : A

Answer Justification :

A primary pollutant is a harmful substance that directly enters the air as a result of human
activities.

Various primary pollutants and their sources in the atmosphere are discussed briefly below

When Coal or natural gas or wood is burnt, carbon dioxide and carbon monoxide are
formed. They are important primary pollutants.

Automobiles also contribute to emission of carbon monoxide.

Nitrogen oxide including nitric oxide and Nitrogen dioxide: its main source includes
vehicular emission, forest fires and volcanoes

it is one of the main causes of smog giving rise to acid rain, when it is transformed into
nitric acid

Sulphur-Dioxide, another primary pollutant, reaches the atmosphere mainly as a


consequence of human activities like burning of coal. Natural Sources such as volcanic
eruption also contribute to its emission.

Hence options 1, 3, 4 and 5 are correct.

Sulphur trioxide is a secondary pollutant.

instacourses.insightsonindia.com 54

https://upscmaterial.online/
Download From - https://upscmaterial.online/

.
Total Marks : 200.00
Test-26 (Subject)
( Insta Prelims Test Series 2023 )

Secondary pollutants result from harmful chemical reaction between two or more air
components.

Sulphur dioxide, a primary pollutant reacts with oxygen in the atmosphere to


form sulphur trioxide which is a secondary pollutant.

The sulphur trioxide can then react with water vapour in the air to form droplets of
sulphuric acid, another secondary pollutant

Ozone is also a secondary pollutant.

Organic aerosol is another secondary pollutant.

Hence options 2 and 6 are incorrect

55. Consider the following statements


1. A Conservation Reserve can be declared by State Government in consultation with local
communities for the purpose of protecting its biodiversity.
2. Marine protected areas in which human activities are strictly regulated than surrounding waters
are defined in India according to IUCN guidelines.
3. Wildlife sanctuaries are areas of natural and geomorphological significance where sustainable
livelihood practices of communities living inside are allowed by law.

Which of the statements given above is/are correct?


A. 1 and 3 only
B. 2 only
C. 2 and 3 only
D. 1, 2 and 3

Correct Answer : D

Answer Justification :

Conservation Reserves are declared for the purpose of protecting landscapes, seascapes,
flora and fauna and their habitat

The rights of people living inside are not affected

Declaration for this purpose can be made by State Government in any area owned by the
government particularly in the areas adjacent to National parks and wildlife sanctuaries

instacourses.insightsonindia.com 55

https://upscmaterial.online/
Download From - https://upscmaterial.online/

.
Total Marks : 200.00
Test-26 (Subject)
( Insta Prelims Test Series 2023 )

and those areas which link one protected area with another

Such declarations are to be made after Consulting local communities.

Hence statement 1 is correct.

Marine Protected Areas are adjacent to oceans, Seas, estuaries or lagoons

In these areas human activities are more strictly regulated than adjoining water bodies

In India, these areas are defined according to IUCN guidelines

They limit anthropogenic activities and exploitation of resources in these areas

Some of the important Marine protected areas in India include:

Gulf of Mannar National Park, Tamil Nadu

Gulf of Kutch Marine National Park, Gujarat

Mahatma Gandhi Marine National Park Andaman and Nicobar Islands

Gahirmatha Marine wildlife sanctuary, Odisha

Hence statement 2 is correct.

Wildlife Sanctuaries are constituted for protection and conservation of wildlife and
environment

These are areas of adequate natural, ecological and geomorphological significance

In a Wildlife Sanctuary, certain rights of people living inside are permitted as


long as they do not harm the wild

Such activities include collection of timber and minor forest products, livestock grazing,
ownership of private land and so on

instacourses.insightsonindia.com 56

https://upscmaterial.online/
Download From - https://upscmaterial.online/

.
Total Marks : 200.00
Test-26 (Subject)
( Insta Prelims Test Series 2023 )

Hence statement 3 is correct.

56. Consider the following statements:


1. Carbon sequestration is the process of capturing and storing atmospheric carbon dioxide.
2. Biological Carbon sequestration is a part of enhanced oil recovery also known as tertiary
recovery.
3. In tertiary recovery, the liquid CO2 is injected into the oil-bearing formation to reduce the
viscosity of the oil.

Which of the statements given above is/are correct?


A. 1 only
B. 2 and 3 only
C. 1 and 3 only
D. 1, 2 and 3

Correct Answer : C

Answer Justification :

Explanation:

Statement 1 is correct: Carbon sequestration is the process of capturing and storing


atmospheric carbon dioxide.

It is one method of reducing the amount of carbon dioxide in the atmosphere with the goal of
reducing global climate change.

Statement 2 is not correct: Geologic carbon sequestration is the process of storing carbon
dioxide (CO2) in underground geologic formations.

The CO2 is usually pressurized until it becomes a liquid, and then it is injected into porous
rock formations in geologic basins.

This method of carbon storage is also sometimes a part of enhanced oil recovery,
otherwise known as tertiary recovery, because it is typically used later in the life of a
producing oil well.

Statement 3 is correct: In enhanced oil recovery, the liquid CO2 is injected into the oil-
bearing formation in order to reduce the viscosity of the oil and allow it to flow more
easily to the oil well.

Biologic carbon sequestration refers to storage of atmospheric carbon in vegetation, soils,

instacourses.insightsonindia.com 57

https://upscmaterial.online/
Download From - https://upscmaterial.online/

.
Total Marks : 200.00
Test-26 (Subject)
( Insta Prelims Test Series 2023 )

woody products, and aquatic environments.

For example, by encouraging the growth of plants—particularly larger plants like


trees—advocates of biologic sequestration hope to help remove CO2 from the atmosphere.

Source:
https://www.usgs.gov/faqs/whats-difference-between-geologic-and-biologic-carbon-s
equestration

57. With reference to the National Initiative on Climate Resilient Agriculture (NICRA),
consider the following statements:
1. The project aims at strategic research on adaptation and mitigation and demonstration of
technologies in farmers’ fields.
2. Under the initiative, Custom hiring centres have been established in the villages to ensure
availability of farm implements for timely operations.

Which of the statements given above is/are correct?


A. 1 only
B. 2 only
C. Both 1 and 2
D. Neither 1 nor 2

Correct Answer : C

Answer Justification :

Explanation:

Indian Council of Agricultural Research (ICAR), Ministry of Agriculture and Farmers Welfare,
Government of India launched a flagship network project ‘National Innovations in Climate
Resilient Agriculture’ (NICRA).

Statement 1 is correct: The project aims at strategic research on adaptation and mitigation,
demonstration of technologies on farmers’ fields and creating awareness among farmers and
other stakeholders to minimize the climatic change impacts on agriculture.

In the strategic research, the main thrust areas covered are

Identifying most vulnerable districts/regions,

Evolving crop varieties and management practices for adaptation and mitigation,

Assessing climate change impacts on livestock, fisheries and poultry and identifying

instacourses.insightsonindia.com 58

https://upscmaterial.online/
Download From - https://upscmaterial.online/

.
Total Marks : 200.00
Test-26 (Subject)
( Insta Prelims Test Series 2023 )

adaptation strategies.

Statement 2 is correct: Custom hiring centres (CHCs) have been established in 121 NICRA
villages to ensure availability of farm implements for timely operations.

Source: https://pib.gov.in/pressreleaseiframepage.aspx?prid=1743354

58. The main purpose of FAME India Scheme is-

A. To reduce the number of vehicles running on diesel and petrol.


B. To increase private participation in the space sector.
C. To provide infrastructural support to semiconductor plants in India.
D. To provide awareness in taking entrepreneurship as a career option.

Correct Answer : B

Answer Justification :

Explanation:

Option (a) is not correct: Government has announced a new organisation, IN-SPACe, part of
reforms to increase private participation in the space sector.

The new Indian National Space Promotion and Authorisation Centre (IN-SPACe), will assess the
needs and demands of private players and explore ways to accommodate these requirements
in consultation with ISRO.

Option (b) is correct: The FAME India scheme has been launched in India to encourage the
citizens of India to buy electric vehicles.

Along with this, citizens will be given subsidy on buying new electric vehicles through this
scheme; its main objective is to promote electric mobility.

Fame Scheme has been started through the central government to control the pollution caused
by the use of petrol and diesel.

Option (c) is not correct: The design-linked incentive (DLI) scheme aims to provide financial
and infrastructural support to companies setting up fabs or semiconductor making plants in
India.

Option (d) is not correct: Entrepreneurship and Skill Development Programme (ESDP)
Scheme aims to motivate different sections of the society such as SC, ST, women, differently
abled, ex-servicemen and people belonging to the BPL category, to take up entrepreneurship as
a career option, by providing motivation and awareness on entrepreneurship, self-employment

instacourses.insightsonindia.com 59

https://upscmaterial.online/
Download From - https://upscmaterial.online/

.
Total Marks : 200.00
Test-26 (Subject)
( Insta Prelims Test Series 2023 )

Source:
www.investindia.gov.in/schemes-msmes-india/entrepreneurship-and-skill-devel

Explained | The DLI scheme and the chip making industry in India - The Hindu

https://indianexpress.com/article/explained/in-space-india-space-missions-private-pa
rticipation-isro-6476532/

Fame India Scheme 2023: Apply Online at fame2.heavyindustry.gov.in


(pmmodischeme.in)

59. With reference to the Long-Term Ecological Observatories (LTEO), consider the
following statements:
1. In India, this initiative, is a network of government bodies and non-profits working together to
build datasets on critical ecosystems.
2. The LTEO landscapes include Western Ghats and Himalayas but exclude Central India due to
the lack of biodiversity hotspots.

Which of the statements given above is/are correct?


A. 1 only
B. 2 only
C. Both 1 and 2
D. Neither 1 nor 2

Correct Answer : A

Answer Justification :

Explanation:

Statement 1 is correct: In India, the government has set up the Long-term Ecological
Observatories initiative, which is a network of government bodies and non-profits working
together to build datasets on critical ecosystems that can help us understand climate change
trends and create effective policy and solutions.

The India - Long Term Ecological Observatories (India - LTEO) programme is a multi-institutional
programme that aims to set up long-term ecological monitoring for different taxa in six
landscapes across India.

Statement 2 is not correct: The LTEO landscapes include Western Ghats, Western
Himalayas, Eastern Himalayas, Central India, North West Arid Zone and Andaman and Nicobar
Islands.

India LTEO includes nine themes including forests, grasslands, soil, herpetofauna, marine
ecosystems, arthropods, freshwater fish birds and movement ecology.

instacourses.insightsonindia.com 60

https://upscmaterial.online/
Download From - https://upscmaterial.online/

.
Total Marks : 200.00
Test-26 (Subject)
( Insta Prelims Test Series 2023 )

Source:
https://www.ncf-india.org/education-and-public-engagement/long-term-ecological-ob
servatories

Sea change: Why India needs lessons from Maldives to tackle climate crisis -
Frontline (thehindu.com)

60. Consider the following statements regarding National Adaptation Fund for Climate
Change (NAFCC):
1. It is a joint Initiative of India in collaboration with United Nations to support concrete adaptation
activities which mitigate the adverse effects of climate change.
2. Only the projects that are related to agriculture and forestry are eligible for funding under
NAFCC.

Which of the statements given above is/are correct?


A. 1 only
B. 2 only
C. Both 1 and 2
D. Neither 1 nor 2

Correct Answer : D

Answer Justification :

Explanation:

Statement 1 is not correct: The National Adaptation Fund for Climate Change (NAFCC) is a
Central Sector Scheme which was set up in the year 2015-16.

The overall aim of NAFCC is to support concrete adaptation activities which mitigate the
adverse effects of climate change.

The activities under this scheme are implemented in a project mode.

Statement 2 is not correct: The projects related to adaptation in sectors such as agriculture,
animal husbandry, water, forestry, tourism etc. are eligible for funding under NAFCC.

National Bank for Agriculture and Rural Development (NABARD) is the National Implementing
Entity (NIE).

Source:
https://vikaspedia.in/energy/environment/climate-change/national-adaptation-fund-f
or-climate-change

instacourses.insightsonindia.com 61

https://upscmaterial.online/
Download From - https://upscmaterial.online/

.
Total Marks : 200.00
Test-26 (Subject)
( Insta Prelims Test Series 2023 )

61. The Bonn Challenge is sometimes seen in news in context of –

A. Restoration of degraded and deforested landscapes


B. Reduction in consumption of ozone-depleting substances
C. Sustainable use of the oceans and marine resources
D. Resurrection biology and reversing extinction in plants

Correct Answer : A

Answer Justification :

Explanation:

Option (a) is correct: The Bonn Challenge is a global goal to bring 150 million hectares of
degraded and deforested landscapes into restoration by 2020 and 350 million hectares by
2030.

It was Launched by the Government of Germany and IUCN in 2011, the Challenge surpassed the
150-million-hectare milestone for pledges in 2017.

The Challenge aligns its work with the Sustainable Development Goals (SDGs), the Aichi
Biodiversity Targets, the Land Degradation Neutrality (LDN) goal, and the Paris Climate Change
Agreement all together providing a roadmap for a sustainable planet.

Source: https://www.bonnchallenge.org/about

62. With reference to Coastal regulation Zones (CRZ), consider the following statements:
1. It refers to the regions in the proximity of India’s shoreline where industrial projects are not
allowed.
2. The CRZ-1 includes the most ecologically sensitive areas allowing rare public utilities projects in
its proximity.
3. According to the CRZ, 2018 notification, eco-tourism activities are permitted in CRZ-1 regions.

Which of the statements given above is/are correct?


A. 1 only
B. 1 and 2 only
C. 2 and 3 only
D. 1, 2 and 3

Correct Answer : C

Answer Justification :

instacourses.insightsonindia.com 62

https://upscmaterial.online/
Download From - https://upscmaterial.online/

.
Total Marks : 200.00
Test-26 (Subject)
( Insta Prelims Test Series 2023 )

Explanation:

Statement 1 is not correct: The coastal regulation zone, or CRZ, 2011, refers to regions in
the proximity of India’s 7000-km-long shoreline where buildings, tourism facilities, industrial
projects, residential facilities etc are highly regulated.

In most cases it begins from the high tide line (HTL) to about 500 metres towards the landward
side.

The zone is subdivided into regions, with varying leeway for infrastructure development,
depending on population and ecological sensitivity.

Statement 2 is correct: The CRZ-I includes the most ecologically sensitive areas and
according to current laws is off-limits for tourism activities and infrastructure development
except for defence, strategic and rare public utilities projects.

Statement 3 is correct: According to the new CRZ, 2018 notification “nature trails and eco-
tourism activities” can be permitted in CRZ-1 regions provided they conform to state-approved
coastal zone management plans.

Source:
https://www.thehindu.com/news/national/centre-proposes-relaxation-of-coastal-regul
ation-zone-
norms/article23590358.ece#:~:text=The%20coastal%20regulation%20zone%2C%20o
r%20CRZ%2C%202011%2C%20refers,to%20about%20500%20metres%20towards%20
the%20landward%20side.

63. Consider the following statements regarding the Solid Waste Management (SWM)
Rules 2016:
1. As per the rules, waste generator has to segregate solid waste into two streams namely bio-
degradable and non-biodegradable waste.
2. Environment Pollution (Prevention and Control) Authority is mandated to enforce provisions of
the Solid Waste Management Rules in all the states.
3. There is a provision of additional central assistance to States and Union territories for
implementing these rules.

Which of the statements given above is/are correct?


A. 1 only
B. 3 only
C. 1 and 2 only
D. 1, 2 and 3

Correct Answer : B

Answer Justification :

instacourses.insightsonindia.com 63

https://upscmaterial.online/
Download From - https://upscmaterial.online/

.
Total Marks : 200.00
Test-26 (Subject)
( Insta Prelims Test Series 2023 )

Explanation:

Statement 1 is not correct: As per Solid Waste Management (SWM) Rules 2016, every waste
generator has to segregate and store solid waste in three separate streams namely bio-
degradable, non-biodegradable and domestic hazardous wastes and handover segregated
wastes to authorised waste pickers or waste collectors as per the direction or notification by the
local authorities from time to time.

The local authorities and village Panchayats of census towns and urban agglomerations are
mandated to establish waste deposition centres for domestic hazardous waste.

Statement 2 is not correct: The State Pollution Control Board or Pollution Control Committee
(SPCB/PCC) is mandated to enforce provisions of the Solid Waste Management Rules in their
State / UT through local bodies in their respective jurisdiction.

The State Governments and Union territory Administration are to constitute a State Level
Advisory Board, as per SWM Rules, to review matters related to implementation of rules and
give advice to State / UT Government for taking measures that are necessary for expeditious
and appropriate implementation of these rules.

Statement 3 is correct: Additional Central Assistance is provided to States and Union


territories for solid waste management, under Swachh Bharat Mission Urban (SBM (U)) and
Swachh Bharat Mission Grameen.

Source: https://www.pib.gov.in/PressReleseDetailm.aspx?PRID=1807658

64. Consider the following statements regarding Compensatory Afforestation


Management and Planning Authority (CAMPA):
1. Works implemented under the CAMPA includes the supply of energy saving devices to the
people living in fringe villages.
2. The Nagar Van Yojana initiated for creation city forests in urban areas over comes under the
purview of CAMPA.

Which of the statements given above is/are correct?


A. 1 only
B. 2 only
C. Both 1 and 2
D. Neither 1 nor 2

Correct Answer : C

Answer Justification :

Explanation:

The Ministry of Environment and constituted the Compensatory Afforestation Fund Management

instacourses.insightsonindia.com 64

https://upscmaterial.online/
Download From - https://upscmaterial.online/

.
Total Marks : 200.00
Test-26 (Subject)
( Insta Prelims Test Series 2023 )

and Planning Authority (CAMPA) for the purpose of management of money collected towards
compensatory afforestation (CA), net present value (NPV) and any other money recoverable
from the user agencies for utilizing forest land for non-forest purposes under the Forest
(Conservation) Act, 1980.

The MOEF, Government of India in 2009 issued guidelines for operating the funds under the
CAMPA.

Statement 1 is correct: Works implemented under the CAMPA include,

Raising of compensatory plantations,

Project specific activities in and around the project area for which forest land has been
diverted (fencing of safety zone, raising of plantation in safety zone, canal plantation,
medicinal plantation, soil and moisture conservation works, supply of energy saving
devices to the people living in fringe villages, etc.) and

Activities for the utilization of NPV (forest consolidation, forest protection, regeneration in
natural forests, wildlife protection and management, infrastructure development, etc.).

Statement 2 is correct: In specific respect to urban areas, the Ministry is implementing the
Nagar Van Yojana initiated for creation of Nagar Van (city forest) in urban areas over a period of
five years under the National Fund of the Compensatory Afforestation Fund Management and
Planning Authority (CAMPA).

Source: https://moef.gov.in/wp-content/uploads/2022/03/Social-Forestry-Schemes.pdf

https://aranya.gov.in/aranyacms/(S(trfi021ori325ndos5dbtivd))/English/IndividualSch
eme.aspx?DbzUuB9w7seZOgXAHWj2pw==

65. Consider the following statements regarding the Coalition Against Wildlife Trafficking
(CAWT):
1. It is an initiative of United Nations Environment Programme.
2. India is a member of this Coalition Initiative.
3. It is exclusively an inter-governmental initiative.

Which of the statements given above is/are not correct?


A. 1 only
B. 2 and 3 only
C. 1, 2 and 3
D. 1 and 3 only

instacourses.insightsonindia.com 65

https://upscmaterial.online/
Download From - https://upscmaterial.online/

.
Total Marks : 200.00
Test-26 (Subject)
( Insta Prelims Test Series 2023 )

Correct Answer : D

Answer Justification :

Explanation:

Statement 1 is not correct: Launched in September 2005, the Coalition against Wildlife
Trafficking (CAWT) led by United States focuses political and public attention on growing threats
to wildlife from poaching and illegal trade.

Statement 2 is correct: India become a member of the US-led Coalition Against Wildlife
Trafficking (CAWT), a coalition that promises to collaborate in the fight against illegal
trade in wildlife and wildlife parts.

“For the first time India and the US have come closer to dealing with issues related to
conservation.

India and the US also agreed to exchanges of park and customs officials regarding public
education about illegal wildlife trade, wildlife and national park management, local
involvement in eco-tourism and human-animal conflict.

Seven major US-based environmental and business groups with global interests and
programmes have joined the coalition.

Statement 3 is not correct: Membership of CAWT is open to governments, NGOs and


corporations committed to combating wildlife trafficking.

Source: https://www.wwfindia.org/?1120/

66. Consider the following statements


1. Electrolysis of water using renewable energy is commonly used for manufacture of green
hydrogen.
2. India’s first pure green hydrogen plant has been commissioned by National Thermal Power
Corporation in Maharashtra.
3. At present 10% of green hydrogen can be blended with Piped Natural Gas in India.

Which of the statements given above is/are correct?


A. 2 only
B. 1 and 3 only
C. 2 and 3 only
D. 1 only

instacourses.insightsonindia.com 66

https://upscmaterial.online/
Download From - https://upscmaterial.online/

.
Total Marks : 200.00
Test-26 (Subject)
( Insta Prelims Test Series 2023 )

Correct Answer : D

Answer Justification :

Green hydrogen is hydrogen produced by splitting water by electrolysis using


power from renewable energy sources.

Recently India has been to generate green hydrogen with the objective of raising non-
Fossil energy capacity to 500 gigawatts by 2030

Hence statement 1 is correct.

India’s first 99% pure green hydrogen plant has been commissioned by Oil India
Limited in eastern Assam’s Jorhat.

The National Thermal Power Corporation has started India’s first green hydrogen blending
operation in Piped natural gas network in Kawas, Surat.

Hence statement 2 is incorrect.

The Petroleum and Natural Gas Regulatory board has given approval for a 5% blending of
green hydrogen with Piped Natural Gas

There are plans to scale up this to 20% in future.

Hence statement 3 is incorrect.

67. Consider the following pairs


Publication : Published by
1. World Social Report : World Bank
2. World Economic : United Nations Situation and Prospects
3. Gender Parity Index : World Economic Forum

How many of the pairs given above are correct?


A. None of the above
B. Only one pair
C. Only two pairs
D. All three pairs

instacourses.insightsonindia.com 67

https://upscmaterial.online/
Download From - https://upscmaterial.online/

.
Total Marks : 200.00
Test-26 (Subject)
( Insta Prelims Test Series 2023 )

Correct Answer : B

Answer Justification :

World Social Report is published by United Nations Department of Economic


and Social Affairs

The report forecasts that global growth will moderately pick up to 2.7% in 2024

Hence pair 1 is incorrect.

The World Social Report is published by United Nations Department of Economic and
Social Affairs

The theme of 2023 report is Leaving no one behind in an aging world

The report highlights that population aging is a defining global trend of the time which
has begun or is expected to begin soon in all countries. People are leaving longer and
more are older than ever before. Improvements in health and survival and reductions in
fertility has driven this momentous shift.

Hence pair 2 is correct.

The Gender Parity Index is released by UNESCO

It is a socio-economic index usually designed to measure the relative access to education


by males and females

Hence pair 3 is incorrect.

68. The Kerch Strait joins

A. Sea of Azov and Black Sea


B. Mediterranean Sea and Red Sea
C. Aral Sea and Caspian Sea
D. Norwegian Sea and Barents Sea

instacourses.insightsonindia.com 68

https://upscmaterial.online/
Download From - https://upscmaterial.online/

.
Total Marks : 200.00
Test-26 (Subject)
( Insta Prelims Test Series 2023 )

Correct Answer : A

Answer Justification :

The Sea of Azov forms a northward extension of the Black Sea linked by the
Strait of Kerch

It links both the seas thus separating Kerch peninsula of Crimea in the West from the
Taman peninsula of Russia’s Krasnodar Krai in the East

It is of much importance for Russia as it is the only direct link between Russia and Crimea
and through it, supply of fuel, food and other goods to Crimea are sent.

It became a major supply route for Russian forces after Russia launched the military
campaign against Ukraine in 2022.

Hence option A is correct.

69. Consider the following statements regarding Trans fatty acids (TFA)
1. Dairy products contain naturally occurring trans fats.
2. TFA increase the number of high-density lipids in human body.
3. Accumulation of TFA in human body is associated with higher risk of development of insulin
resistance.

Which of the statements given above is/are correct?


A. 1 and 2 only
B. 2 only
C. 1 and 3 only
D. 3 only

instacourses.insightsonindia.com 69

https://upscmaterial.online/
Download From - https://upscmaterial.online/

.
Total Marks : 200.00
Test-26 (Subject)
( Insta Prelims Test Series 2023 )

Correct Answer : C

Answer Justification :

Trans fatty acids are unsaturated fatty acids that are of two types:

Naturally occurring trans-fat are found in some dairy and meat products

Industrially produced trans-fat add hydrogen to liquid vegetable oils to make them more
solid. It is found in packaged foods, packed goods, cooking oils and spreads.

Hence statement 1 is correct.

High density lipids or HDL are the good cholesterol that help in protection
against heart diseases.

TFA reduce HDL and raise the total cholesterol level in human body and the pose a
higher risk of heart disease than saturated fats.

Hence statement 2 is incorrect.

TFA is associated with higher risk of developing obesity, type 2 diabetes, metabolic
syndrome, insulin resistance, infertility and certain types of cancer

They can also lead to compromised foetal development causing harm to the foetus.

Hence statement 3 is correct.

70. Recently, a new species of bamboo dwelling thick-thumbed bat has been reported for
the first time from South Asia, from

A. Arunachal Pradesh
B. Mizoram
C. Meghalaya
D. Assam

Correct Answer : C

instacourses.insightsonindia.com 70

https://upscmaterial.online/
Download From - https://upscmaterial.online/

.
Total Marks : 200.00
Test-26 (Subject)
( Insta Prelims Test Series 2023 )

Answer Justification :

Recently a new species of bamboo dwelling bat has been found in Ri Bhoi
district of Meghalaya, India

It is the first report of thick-thumbed bat not only from India but also from South Asia

The species found near the forested patch of Nongkhyllem wildlife sanctuary has been
named Glischropus meghalayanus

The scientists from Zoological Survey of India collected the specimen from the forest.

This newly discovered species is small in size and has a dark brown colour with Sulphur
yellow belly.

Bamboo dwelling bats are particular kinds of bat who live in the internodes of bamboo
plant.

They have specialised morphological characters that help them adapt the life inside a
bamboo plant.

This species of bat is thick thumbed, that is they have typical fleshy pads on the thumb
and soles of the feet that help them to crawl over smooth surfaces of bamboo internodes.

Hence option C is correct.

71. The Aichi biodiversity targets often talked about in news were established by the-

A. United Nations Convention of Biological Diversity


B. United Nations Biodiversity Conference (COP15)
C. Cancun Adaptation Framework
D. International Union for Conservation of Nature

Correct Answer : A

Answer Justification :

Option (a) is correct: The Aichi biodiversity targets were established by the United Nations
Convention of Biological Diversity and consist of 20 specific targets to address and mitigate

instacourses.insightsonindia.com 71

https://upscmaterial.online/
Download From - https://upscmaterial.online/

.
Total Marks : 200.00
Test-26 (Subject)
( Insta Prelims Test Series 2023 )

biodiversity loss across the globe.

20 biodiversity targets were set by the UN Convention of Biological Diversity in 2011 to offer a
framework in which the world and individual countries can address the growing rates of
biodiversity loss.

These 20 biodiversity targets work in tandem to meet five overarching goals:

Address the underlying causes of biodiversity loss by making it a priority in government


agendas;
Curtail the direct pressures on biodiversity and promote sustainable use;
Safeguarding ecosystems, species and diversity;
Highlight and improve the benefits we derive from ecosystem services;
Expand efforts in knowledge management and capacity building.

Source: https://earth.org/what-are-the-aichi-biodiversity-targets/

72. India’s new Draft Arctic Policy aims at-


1. Expanding scientific research
2. Sustainable tourism
3. Oil and gas exploration
4. Waste disposal practices

Which of the statements given above is/are correct?


A. 1 and 4 only
B. 1, 2 and 3 only
C. 1, 2 and 4 only
D. 1, 2, 3 and 4

Correct Answer : B

Answer Justification :

Option (b) is correct: India has unveiled a new draft ‘Arctic’ policy that, among other things,
commits to expanding scientific research, “sustainable tourism” and mineral oil and gas
exploration in the Arctic region.

Other objectives of the policy include putting in place Arctic related programmes for
mineral/oil and gas exploration in petroleum research institutes and encouraging tourism
and hospitality sectors in building specialised capacities and awareness to engage with
Arctic enterprises.
Five Arctic littoral states — Canada, Denmark (Greenland), Norway, Russia and the USA
(Alaska) — and three other Arctic nations — Finland, Sweden and Iceland — form the
Arctic Council.
India launched its first scientific expedition to the Arctic in 2007 and set up a research

instacourses.insightsonindia.com 72

https://upscmaterial.online/
Download From - https://upscmaterial.online/

.
Total Marks : 200.00
Test-26 (Subject)
( Insta Prelims Test Series 2023 )

station ‘Himadri’ in the international Arctic research base at Ny-Ålesund in Spitsbergen,


Svalbard, Norway

Source:
https://www.thehindu.com/news/national/india-to-expand-research-tourism-in-arctic/
article33636563.ece

73. Consider the following statements regarding the Vienna Convention:

1. It speaks to the enormity of ozone depletion and the willingness of countries to work together to
solve it.
2. It essentially requires countries to take control actions to protect the ozone layer.

Which of the statements given above is/are correct?


A. 1 only
B. 2 only
C. Both 1 and 2
D. Neither 1 nor 2

Correct Answer : A

Answer Justification :

By 1985, the world had already seen advancements in the scientific understanding of ozone
depletion and its impacts on human health and the environment.

It was then that the Vienna Convention for the Protection of the Ozone Layer was created in
response.

Statement 1 is correct: The Vienna Convention was the first convention of any kind to be
signed by every country involved, taking effect in 1988 and reaching universal ratification in
2009. This speaks to the enormity of ozone depletion at the time and the willingness of
countries around the world to work together to solve it.

This agreement is a framework convention that lays out principles agreed upon by many
parties.

Statement 2 is not correct: It does not, however, require countries to take control actions to
protect the ozone layer. This would come later in the form of the Montreal Protocol.

Source: https://ozone.unep.org/treaties/vienna-convention

74. With reference to Ramsar Convention on Wetlands, consider the following


statements:

instacourses.insightsonindia.com 73

https://upscmaterial.online/
Download From - https://upscmaterial.online/

.
Total Marks : 200.00
Test-26 (Subject)
( Insta Prelims Test Series 2023 )

1. It provides single most global framework for intergovernmental cooperation on wetland issues.
2. At the time of joining, each Contracting Party must designate at least one wetland site within
their territory as a wetland of International Importance.
3. The Convention's Strategic Plan acknowledges importance of partnerships in implementing the
Convention.

Which of the statements given above is/are correct?


A. 1 only
B. 1 and 2 only
C. 2 and 3 only
D. 1, 2 and 3

Correct Answer : D

Answer Justification :

Statement 1 is correct: The Convention on Wetlands is the oldest of the modern global
intergovernmental environmental agreements.

The Convention on Wetlands provides the single most global framework for intergovernmental
cooperation on wetland issues.

The treaty was negotiated through the 1960s by countries and non - governmental
organizations concerned about the increasing loss and degradation of wetland habitat for
migratory waterbirds.

Statement 2 is correct: At the time of joining the Convention, each Contracting Party must
designate at least one wetland site within their territory for inclusion in the List of Wetlands of
International Importance

The Convention works with a diverse global network of partners to meet its objectives.

They range from other global Conventions and agencies, to international and national non-
governmental organizations and prominent private companies.

Statement 3 is correct: The Convention's Strategic Plan 2016-2024 acknowledges the


importance of partnerships in enhancing the implementation of the Convention

Source: https://www.ramsar.org/about/partnerships

75. Which of the following specific areas are encompassed in the missions of National
Action Plan on climate change (NAPCC)?
1. Enhanced Energy Efficiency
2. Deep Sea Mining
3. Green India
4. Sustainable Agriculture

instacourses.insightsonindia.com 74

https://upscmaterial.online/
Download From - https://upscmaterial.online/

.
Total Marks : 200.00
Test-26 (Subject)
( Insta Prelims Test Series 2023 )

Select the correct answer using the code given below:


A. 1 and 2 only
B. 1, 3 and 4 only
C. 2 and 4 only
D. 1, 2, 3 and 4

Correct Answer : B

Answer Justification :

Option (b) is correct: The National Action Plan on climate change (NAPCC) encompasses eight
missions in the specific areas of Solar Energy, Enhanced Energy Efficiency, Sustainable Habitat,
Water, Sustaining the Himalayan Eco-system, Strategic Knowledge for Climate Change, Green
India, and Sustainable Agriculture.

These specific areas tackle core issues pertaining to climate change, outlining steps to
simultaneously advance India’s development and climate change related-objectives of
adaptation and mitigation.

NAPCC is guided by the principles –

Protecting the poor and vulnerable sections of society through an inclusive and
sustainable development strategy, sensitive to climate change;
Achieving national growth through ecological sustainability
Devising efficient and cost-effective strategies for end use Demand Side Management
Deploying appropriate technologies for both adaptation and mitigation of greenhouse
gases emissions
Engineering new and innovative forms of market, regulatory and voluntary mechanisms
to promote sustainable development
Effecting implementation of programmes by including civil society and local government
institutions and through public-private partnership; and
Welcoming international cooperation for research, development, sharing and transfer of
technologies.

Source: https://pib.gov.in/PressReleaseIframePage.aspx?PRID=1810566

76. Which one of the following statements best describes Kigali Agreement?

A. It calls for a gradual reduction in the consumption and production of the


hydrofluorocarbons.
B. It helps countries to achieve sustainable impact on criminal justice and corruption
prevention.
C. It supports parties in the implementation of obligations under Rotterdam and Stockholm
Conventions.
D. It is a standardized way of communicating the environmental impacts of acidification and

instacourses.insightsonindia.com 75

https://upscmaterial.online/
Download From - https://upscmaterial.online/

.
Total Marks : 200.00
Test-26 (Subject)
( Insta Prelims Test Series 2023 )

eutrophication.

Correct Answer : A

Answer Justification :

Option (a) is correct: The Kigali Amendment calls for a gradual reduction in the consumption
and production of hydrofluorocarbons (“HFCs”), which are potent greenhouse gases.

Its global implementation should avoid as much as half a degree Celsius of warming by the end
of the century.

The Montreal Protocol, which also regulates the production and consumption of ozone depleting
substances, is one of the most successful international environmental agreements.

Source: U.S. Ratification of the Kigali Amendment - United States Department of


State

77. Which of the following statements is/are not correct about the International Blue
Carbon Initiative?
1. The Initiative is a global program and works to restore coastal ecosystems for their role in
reducing impacts of global climate change.
2. The Initiative focuses on mangroves, salt marshes and seagrasses which are found on every
continent of the world.

Select the correct answer using the code given below:


A. 1 only
B. 2 only
C. Both 1 and 2
D. Neither 1 nor 2

Correct Answer : B

Answer Justification :

Statement 1 is correct: The International Blue Carbon Initiative is a coordinated, global


program focused on mitigating climate change through the conservation and restoration of
coastal and marine ecosystems.

Blue carbon is the carbon stored in coastal and marine ecosystems.

The Blue Carbon Initiative currently focuses on carbon in coastal ecosystems - mangroves, tidal
marshes and seagrasses.

These ecosystems sequester and store large quantities of blue carbon in both the plants and

instacourses.insightsonindia.com 76

https://upscmaterial.online/
Download From - https://upscmaterial.online/

Total Marks : 200.00


Test-26 (Subject)
( Insta Prelims Test Series 2023 )

the sediment below. For example, over 95% of the carbon in seagrass meadows is stored in the
soils.

Statement 2 is not correct: The Blue Carbon Initiative focuses on mangroves, salt marshes
and seagrasses, which are found on every continent except Antarctica.

Source: https://www.thebluecarboninitiative.org/

78. With reference to the REDD+, consider the following statements:


1. It is a framework created by the UNFCCC Conference of the Parties.
2. It aims to fulfil the commitment to climate actions in the forest sector.
3. The Parties of the Paris Agreement do not encourage its implementation.

Which of the statements given above is/are correct?


A. 1 only
B. 2 only
C. 1 and 2 only
D. 2 and 3 only

Correct Answer : C

Answer Justification :

Statement 1 is correct: REDD+ is a framework created by the UNFCCC Conference of the


Parties (COP) to guide activities in the forest sector that reduces emissions from deforestation
and forest degradation, as well as the sustainable management of forests and the conservation
and enhancement of forest carbon stocks in developing countries.

Statement 2 is correct: It aims at the implementation of activities by national governments


to reduce human pressure on forests that result in greenhouse gas emissions at the national
level, but as an interim measure also recognizes subnational implementation.

The implementation of REDD+ activities is voluntary and depends on the national


circumstances, capacities and capabilities of each developing country and the level of support
received.

Statement 3 is not correct: REDD+ is also recognized in Article 5 of the Paris Agreement,
where Parties re-iterated the encouragement to implement REDD+ activities, and that these
should be an integral element of the Paris Agreement.

Source: https://unfccc.int/topics/land-use/workstreams/redd/what-is-redd

79. Consider the following statements:


1. Any process that uses fossil fuels releases more carbon into the atmosphere than carbon sinks

instacourses.insightsonindia.com 77

https://upscmaterial.online/
Download From - https://upscmaterial.online/

Total Marks : 200.00


Test-26 (Subject)
( Insta Prelims Test Series 2023 )

can absorb.
2. Cattle farming contributes to deforestation but does not contribute in the depletion of carbon
sinks.
3. Both the oceans as well as the forests act as the natural carbon sink in the world.

Which of the statements given above is/are correct?


A. 1 only
B. 2 and 3 only
C. 1 and 3 only
D. 1, 2 and 3

Correct Answer : C

Answer Justification :

Statement 1 is correct: Any process that uses fossil fuels such as burning coal to generate
electricity releases more carbon into the atmosphere than carbon sinks can absorb. Cattle
farming also releases a lot of carbon into the atmosphere.

Statement 2 is not correct: It also contributes to deforestation, depleting the planet of its
carbon sinks;

Statement 3 is correct: Aside from the aforementioned oceans being the main natural carbon
sink in the world, forests are also significant carbon sinks examples as well.

According to a report, forests absorb twice as much carbon as they release each year,
absorbing a net 7.6 billion metric tonnes of carbon dioxide annually.

Source: https://earth.org/carbon-sinks/

80. Consider the following statements regarding Indian Antarctic Bill, 2022:
1. It prohibits private expeditions to Antarctica without written authorisation by a member country.
2. It extends the jurisdiction of Indian courts to Antarctica but has no penal provision for crimes by
the foreign citizens on the continent.
3. It bars the testing of nuclear devices but has no provisions against disposing waste in
Antarctica.

Which of the statements given above is/are correct?


A. 1 only
B. 1 and 2 only
C. 2 and 3 only
D. 1, 2 and 3

Correct Answer : A

instacourses.insightsonindia.com 78

https://upscmaterial.online/
Download From - https://upscmaterial.online/

Total Marks : 200.00


Test-26 (Subject)
( Insta Prelims Test Series 2023 )

Answer Justification :

Indian Antarctic Bill, 2022, lays out a structure for government officials to inspect a vessel and
conduct checks of research facilities.

The draft also directs the creation of a fund called the Antarctic fund that will be used for
protecting the Antarctic environment.

Statement 1 is correct: The private tours and expeditions to Antarctica would be prohibited
without a permit or the written authorisation by a member country.

Statement 2 is not correct: The Bill extends the jurisdiction of Indian courts to Antarctica and
lays out penal provision for crimes on the continent by Indian citizens, foreign citizens who are a
part of Indian expeditions, or are in the precincts of Indian research stations.

Statement 3 is not correct: The Bill prohibits mining, dredging and activities that threaten
the pristine conditions of the continent.

It bans any person, vessel or aircraft from disposing waste in Antarctica and bars the testing of
nuclear devices.

Source:
https://www.thehindu.com/sci-tech/science/the-indian-antarctic-bill-and-its-various-p
rovisions/article65293882.ece

81. Prosopis Juliflora, sometime seen in the news, is native to

A. Mexico
B. Africa
C. Asia
D. Australia

Correct Answer : A

Answer Justification :

Prosopis Juliflora is a shrub or small tree in the family Fabaceae, a kind of mesquite. It is native
to Mexico, South America and the Caribbean. It has become established as an invasive weed in
Africa, Asia, Australia and elsewhere.

It is a contributing factor to continuing transmission of malaria, especially during dry periods


when sugar sources from native plants are largely unavailable to mosquitoes.

https://www.cabi.org/news-article/invasive-tree-prosopis-juliflora-seriously-reduces-water-resour
ces-in-ethiopia-costing-rural-livelihoods-new-research-reveals/

instacourses.insightsonindia.com 79

https://upscmaterial.online/
Download From - https://upscmaterial.online/

Total Marks : 200.00


Test-26 (Subject)
( Insta Prelims Test Series 2023 )

82. Which of the following is/are the objectives of National Green Hydrogen Mission
1. Development of green hydrogen production capacity of at least 5 MMT by 2030.
2. Cumulative reduction in fossil fuel imports over Rs. One lakh crore by 2030.

Which of the statements given above is/are correct?


A. 1 only
B. 2 only
C. Both 1 and 2
D. Neither 1 nor 2

Correct Answer : C

Answer Justification :

Both the statements are correct.

The Union Cabinet, chaired by the Hon’ble Prime Minister Shri Narendra Modi, has approved
National Green Hydrogen Mission. The initial outlay for the Mission will be Rs19,744 crores,
including an outlay of Rs.17,490 crores for the SIGHT-Programme, Rs.1,466 crores for pilot
projects, Rs.400 crores for R&D, and Rs. 388 crores towards other Mission components. MNRE
will formulate the scheme guidelines for implementation of the respective components.

The Mission will result in the following likely outcomes by 2030:

Development of green hydrogen production capacity of at least 5 MMT (Million


Metric Tonne) per annum with an associated renewable energy capacity
addition of about 125 GW in the country
Over Rs. Eight lakh crores in total investments
Creation of over Six lakh jobs.
Cumulative reduction in fossil fuel imports over Rs. One lakh crore.
Abatement of nearly 50 MMT of annual greenhouse gas emissions.

https://pib.gov.in/PressReleasePage.aspx?PRID=1888547

83. Consider the following statements


1. Madhya Pradesh has highest leopard population in India
2. Karnataka has highest tiger population in India
3. Assam has highest elephant population in India.

Which of the statements given above is/are not correct?


A. 1 and 2 only
B. 2 and 3 only
C. 1 and 3 only
D. 1, 2 and 3

instacourses.insightsonindia.com 80

https://upscmaterial.online/
Download From - https://upscmaterial.online/

Total Marks : 200.00


Test-26 (Subject)
( Insta Prelims Test Series 2023 )

Correct Answer : B

Answer Justification :

Madhya Pradesh regained the top position in the 2018 tiger census after it was found to be
home to 526 felines, two more than Karnataka (524).

instacourses.insightsonindia.com 81

https://upscmaterial.online/
Download From - https://upscmaterial.online/

Total Marks : 200.00


Test-26 (Subject)
( Insta Prelims Test Series 2023 )

84. Consider the following statements regarding Climate Action Tracker (CAT)
1. It is an independent scientific project that tracks government climate action and measures it
against the globally agreed Paris Agreement.
2. It covers all the CO2 emitting countries in the world.
3. It has received generous support from foundations and governments.

Which of the statements given above are correct?


A. 1 only
B. 1 and 3 only
C. 3 only
D. 2 and 3 only

Correct Answer : B

Answer Justification :

The Climate Action Tracker is an independent scientific project that tracks government climate
action and measures it against the globally agreed Paris Agreement aim of "holding warming
well below 2°C, and pursuing efforts to limit warming to 1.5°C." A collaboration of two
organisations, Climate Analytics and New Climate Institute, the CAT has been providing this
independent analysis to policymakers since 2009.

instacourses.insightsonindia.com 82

https://upscmaterial.online/
Download From - https://upscmaterial.online/

Total Marks : 200.00


Test-26 (Subject)
( Insta Prelims Test Series 2023 )

CAT covers all the biggest emitters and a representative sample of smaller emitters
covering about 85% of global emissions and approximately 70% of global population. Hence
Statement 2 is incorrect.

The national actions we track are:

Effect of climate policies and action on emissions: The policies a government has
implemented or enacted and how these are likely to affect national emission over the
time period to 2030, and where possible beyond.
Impact of pledges, targets and NDCs on national emissions over the time period to
2030, and where possible beyond.
Comparability of effort against countries’ fair share and modelled domestic
pathways: Whether a government is doing its “fair share” compared with others towards
the global effort to limit warming consistent with the Paris Agreement, and whether its
mitigation efforts on its own territory are in line with global least cost pathways.

The Climate Action Tracker is made possible due to generous support from foundations and
governments, including the European Climate Foundation (aviation, shipping, Thailand, Iran,
Nigeria, Colombia, Germany, UK, Vietnam, Kenya, governments reactions to the COVID-19
pandemic and climate governance) and the German Ministry for Environment, Nature
Conservation and Nuclear Safety (BMU) via the International Climate Initiative (other elements).

https://climateactiontracker.org/about/

85. Consider the following statements


1. A cheetah has a small and well-round head shape whereas a leopard has a more elongated
head with dark blackish spots on the muzzle.
2. A cheetah has more of an amber-coloured eye as opposed to a leopard which has more of a
green-blue coloured eye.
3. Cheetah are taller and slender in build compared to a leopard which are stronger and bulkier.

Which of the statements given above is/are correct?


A. 1 and 3 only
B. 1, 2 and 3
C. 2 and 3 only
D. 2 and 3 only

Correct Answer : B

Answer Justification :

Cheetahs and Leopards are both part of the “big cat” family. However, the two are very
different cats that can be found across Africa and if your lucky can be spotted during a safari
trip. There are several physical and behavioural characteristics that differ between a cheetah
and a leopard. Yet many people cannot tell them apart. After reading this article you will

instacourses.insightsonindia.com 83

https://upscmaterial.online/
Download From - https://upscmaterial.online/

Total Marks : 200.00


Test-26 (Subject)
( Insta Prelims Test Series 2023 )

become an expert in telling the difference between a cheetah and a leopard as well as
learn some interesting facts about both that might come in handy at your next trivia game
night.

THE FUR COAT:

First, a key difference is in the fur coat of a cheetah and a leopard. It’s all in the spots! In simple
terms, cheetahs have spots and leopards have rosettes. The spots on a cheetah are
individual solid round or oval black spots all over its body. Leopard however, have a
more complex pattern consisting of rosettes which are irregularly shaped spots that are
grouped giving it rose-like markings. This pattern on the leopard helps with camouflage while a
leopard hunts by simulating grass movement and shadows. Another small difference is the fur
coat colour between a cheetah and a leopard.

THE FACE & HEAD:

Another key variation between a cheetah and a leopard is in the facial markings. Cheetah
have black “tear marks” that run down from the inner corner of their eyes down the
sides of their nose to their mouth whereas a leopard has no “tear” line. This black line on a
cheetah face helps with reflecting the sun glare during the daytime while it is hunting. Basically,
a cheetah’s own version of sunglasses. The head shape of a cheetah versus a leopard are also
different. A cheetah has a small and well-round head shape whereas a leopard has a
more elongated head with dark blackish spots on the muzzle. Also, Cheetahs have
smaller teeth and jaws with a larger nasal cavity to allow rapid breathing while reaching top
speeds versus a leopard which has a stronger jaw and teeth that can break through bones and
allow them to drag their prey up trees.

THE EYES:

A cheetah and a leopard eye colour also differ. A cheetah has more of an amber coloured
eye as opposed to a leopard which has more of a green-blue coloured eye.

SIZE & BODY SHAPE:

instacourses.insightsonindia.com 84

https://upscmaterial.online/
Download From - https://upscmaterial.online/

Total Marks : 200.00


Test-26 (Subject)
( Insta Prelims Test Series 2023 )

The size and body shape of a cheetah also has key distinctions versus a leopard. Leopards are
actually the smallest of the cats but is stronger and bulkier than the cheetah. Cheetahs are
taller and slender in build compared to a leopard which are stronger and bulkier. A
cheetah is built for speed and is the fastest land mammal, reaching speeds of up to 113km/h
(70mph). Cheetahs can reach top speeds due to their long bodies, thin stomach, high chest,
long legs for effective acceleration, and flexible spines to allow for rapid changes in direction
when chasing prey. Leopards have short and muscular legs for effective tree-climbing.
Leopards also have bigger muscles around the shoulders and neck which allow them to drag
their prey up trees. Male cheetah weight around 54kg and female cheetah weight around 43kg.
Male leopard weights about 60-70kg and female leopard weight around 30-40kg.

https://www.safariventures.com/whats-the-difference-cheetah-vs-leopard/

86. Consider the following statements regarding Forest Advisory Committee (FAC)
1. It is a non-statutory body.
2. It comes under Ministry of Tribal Affairs.

Which of the statements given above is/are correct?


A. 1 only
B. 2 only
C. Both 1 and 2
D. Neither 1 nor 2

Correct Answer : D

Answer Justification :

Both the statements are incorrect.

Forest Advisory Committee (FAC)

It is a statutory body which was constituted by the Forest (Conservation) Act 1980. It
comes under the Ministry of Environment, Forests & Climate Change (MoEF&CC).

It considers questions on the diversion of forest land for non-forest


uses such as mining, industrial projects, townships and advises the government on the issue of
granting forest clearances. However, its role is advisory.

https://www.thehindu.com/sci-tech/energy-and-environment/fac-seeks-revised-proposal-for-etali
n-hydro-project-in-arunachal-pradesh/article66396079.ece

87. Indian Star Tortoises, sometime seen in the news, is protected under which schedule
of the Wildlife Protection Act, 1972

instacourses.insightsonindia.com 85

https://upscmaterial.online/
Download From - https://upscmaterial.online/

Total Marks : 200.00


Test-26 (Subject)
( Insta Prelims Test Series 2023 )

A. Schedule I
B. Schedule II
C. Schedule III
D. Schedule IV

Correct Answer : D

Answer Justification :

Indian star tortoises are easily recognizable by their beautifully star-patterned shells.

Typically, shy and small, relative to other tortoises, they prefer natural sunlight and life
outdoors but can adapt to indoor living with adequate heat, lighting, and humidity. Indian star
tortoises are native to the arid climates in India, Pakistan, and Sri Lanka, and are accustomed to
monsoon seasons

Indian Star Tortoises are protected under Schedule IV of WPA,1972. It is listed as


Vulnerable.

https://www.thehindu.com/sci-tech/energy-and-environment/indian-star-tortoise-faces-twin-chall
enges-of-habitat-loss-and-genetic-diversity-finds-study/article66397486.ece

88. Consider the following statements


1. Nubra Valley is to the north of Leh.
2. Nubra river is the right tributary of Indus.
3. Indus River flows between Ladakh and Karakoram range

Which of the statements given above is/are correct?


A. 1 only
B. 1 and 2 only
C. 3 only
D. 1, 2 and 3

Correct Answer : B

Answer Justification :

instacourses.insightsonindia.com 86

https://upscmaterial.online/
Download From - https://upscmaterial.online/

Total Marks : 200.00


Test-26 (Subject)
( Insta Prelims Test Series 2023 )

89. Consider the following statements regarding Ammonia


1. It is the basic building block of all nitrogen fertilizer.
2. It has no odour
3. It can be used as a refrigerant gas, to purify water supplies, and in the manufacture of plastics,
explosives, fabrics, pesticides, dyes and other chemicals.

instacourses.insightsonindia.com 87

https://upscmaterial.online/
Download From - https://upscmaterial.online/

Total Marks : 200.00


Test-26 (Subject)
( Insta Prelims Test Series 2023 )

Which of the statements given above is/are correct?


A. 1 and 2 only
B. 2 and 3 only
C. 1 and 3 only
D. 1, 2 and 3

Correct Answer : C

Answer Justification :

Ammonia (NH3) is the basic building block of all nitrogen fertilizer. It is produced
primarily through the Haber-Bosch Process, i.e ammonia production from hydrogen (H2) and
nitrogen (N2) through a high pressure (150-300 bar) exothermic catalytic reaction at 350-500
°C.

Ammonia is a colourless highly irritating gas with a sharp suffocating odour. It


dissolves easily in water to form ammonium hydroxide solution which can cause irritation and
burns. Ammonia gas is easily compressed and forms a clear, colourless liquid under pressure.
Hence Statement 2 is incorrect.

About 80% of the ammonia produced in industry is used in agriculture as fertilizer. Ammonia is
also used as a refrigerant gas, to purify water supplies, and in the manufacture of
plastics, explosives, fabrics, pesticides, dyes and other chemicals. It is found in many
household and industrial-strength cleaning solutions. Cleaning solutions for industrial use
contain higher concentrations of ammonia and can quickly cause irritation and burns.

https://www.health.ny.gov/environmental/emergency/chemical_terrorism/ammonia_general.htm

90. Access, Watch and Reserve (AWaRe), sometime seen in the news, is related to

A. Wildlife Conservation
B. Access to drinking water and sanitation
C. Antimicrobial Resistance
D. Climate Change Initiatives

Correct Answer : C

Answer Justification :

The AWaRe Classification of antibiotics was developed in 2017 by the WHO Expert Committee
on Selection and Use of Essential Medicines as a tool to support antibiotic stewardship efforts at
local, national and global levels, Antibiotics are classified into three groups, Access, Watch
and Reserve, taking into account the impact of different antibiotics and antibiotic
classes on antimicrobial resistance, to emphasize the importance of their

instacourses.insightsonindia.com 88

https://upscmaterial.online/
Download From - https://upscmaterial.online/

Total Marks : 200.00


Test-26 (Subject)
( Insta Prelims Test Series 2023 )

appropriate use. The 2021 update of the AWaRe classification includes an additional 78
antibiotics not previously classified, bringing the total to 258.

It is a useful tool for monitoring antibiotic consumption, defining targets and monitoring the
effects of stewardship policies that aim to optimize antibiotic use and curb antimicrobial
resistance. The WHO 13th General Programme of Work 2019–2023 includes a country-level
target of at least 60% of total antibiotic consumption being Access group antibiotics.

https://www.who.int/publications/i/item/2021-aware-classification

91. Which of the following countries is/are the members of Organization of Turkic States
1. Turkey
2. Afghanistan
3. Tajikistan
4. Pakistan

Select the correct answer using the code given below


A. 1 only
B. 2 and 3 only
C. 1 and 3 only
D. 2 and 4 only

Correct Answer : A

Answer Justification :

The Organization of Turkic States (OTS), formerly called the Turkic Council or the Cooperation
Council of Turkic Speaking States, is an international organization comprising prominent
independent Turkic countries: Azerbaijan, Kazakhstan, Kyrgyzstan, Turkey and
Uzbekistan.

It is an intergovernmental organization whose overarching aim is promoting comprehensive


cooperation among Turkic-speaking states. First proposed by Kazakh President Nursultan
Nazarbayev in 2006, it was founded on 3 October 2009, in Nakhchivan. The General Secretariat
is in Istanbul.

The organization was established on 3 October 2009 as the Cooperation Council of the Turkic
Speaking States (Turkic Council), by the Nakhchivan Agreement signed among Azerbaijan,
Kazakhstan, Kyrgyzstan, and Turkey.

https://en.wikipedia.org/wiki/Organization_of_Turkic_States

92. Consider the following statements

instacourses.insightsonindia.com 89

https://upscmaterial.online/
Download From - https://upscmaterial.online/

Total Marks : 200.00


Test-26 (Subject)
( Insta Prelims Test Series 2023 )

1. The European Parliament represents the citizens of EU countries and is directly elected by
them.
2. The heads of state or government of the EU countries meet, as the European Council, to define
the general political direction and priorities of the European Union.

Which of the statements given above is/are correct?


A. 1 only
B. 2 only
C. Both 1 and 2
D. Neither 1 nor 2

Correct Answer : C

Answer Justification :

Both the statements are correct.

The European Parliament represents the citizens of EU countries and is directly


elected by them. It takes decisions on European laws jointly with the Council of the
European Union. It also approves the EU budget. It runs a network of liaison offices
in EU capitals, London, Edinburgh and Washington D.C.

The heads of state or government of the EU countries meet, as the European Council,
to define the general political direction and priorities of the European Union. The
European Council is chaired by a president who is elected for a 2.5-year term,
renewable once. It does not adopt laws except for possible EU Treaty amendments.

https://indianexpress.com/article/world/european-union-10th-package-russia-sanctio
ns-8465645/

93. Consider the following statements regarding European Investment Bank


1. The Bank borrows money on capital markets and lends it on favourable terms to projects that
support EU objectives.
2. The EIB is the majority shareholder of the European Investment Fund (EIF), which provides
funding to small and medium-sized enterprises (SMEs) through venture capital and risk finance
instruments.

Which of the statements given above is/are correct?


A. 1 only
B. 2 only
C. Both 1 and 2
D. Neither 1 nor 2

Correct Answer : C

instacourses.insightsonindia.com 90

https://upscmaterial.online/
Download From - https://upscmaterial.online/

Total Marks : 200.00


Test-26 (Subject)
( Insta Prelims Test Series 2023 )

Answer Justification :

Both the statements are correct.

The Bank borrows money on capital markets and lends it on favourable terms to projects that
support EU objectives. About 90 % of loans are made within the EU. None of the money
comes from the EU budget.

The EIB provides 3 main types of products and services:

Lending – about 90 % of its total financial commitment. The Bank lends to clients of all
sizes to support growth and jobs, and this support often helps to attract other investors
'Blending' - allowing clients to combine EIB financing with additional investment
Advising and technical assistance - maximising value for money

The EIB is the majority shareholder of the European Investment Fund (EIF), which
provides funding to small and medium-sized enterprises (SMEs) through venture
capital and risk finance instruments. Other shareholders are the European Commission and
financial institutions from across Europe. Established in 1994, the Fund is active in all EU
countries, prospective member countries, Liechtenstein and Norway.

https://european-union.europa.eu/institutions-law-budget/institutions-and-bodies/institutions-an
d-bodies-profiles/eib_en

94. Consider the following statements regarding Schengen area and EU


1. Schengen area allows people and businesses to travel and operate without border checks.
2. It doesn’t include Non – EU countries.
3. The EU budget is funded by a percentage of each member country’s gross national income.

Which of the statements given above is/are correct?


A. 3 only
B. 2 and 3 only
C. 1 and 3 only
D. 1, 2 and 3

Correct Answer : C

Answer Justification :

Since 1985, the Schengen area allows people and businesses to travel and operate without
border checks. It began in June 1985 with 5 countries. Today it contains 23 EU
countries and 4 non-EU countries. Hence Statement 2 is incorrect.

Schengen countries have also strengthened security at their common external border.

instacourses.insightsonindia.com 91

https://upscmaterial.online/
Download From - https://upscmaterial.online/

Total Marks : 200.00


Test-26 (Subject)
( Insta Prelims Test Series 2023 )

The EU budget is funded by a percentage of each member country’s gross national income and
other income sources.

https://european-union.europa.eu/principles-countries-history/key-facts-and-figures/structure_en

95. Which of the following places is/are located in Turkey?


1. Malatya
2. Ekinozu
3. Hama

Select the correct answer using the code given below


A. 1 and 3 only
B. 2 only
C. 1 and 2 only
D. 3 only

Correct Answer : C

Answer Justification :

instacourses.insightsonindia.com 92

https://upscmaterial.online/
Download From - https://upscmaterial.online/

Total Marks : 200.00


Test-26 (Subject)
( Insta Prelims Test Series 2023 )

https://www.thehindu.com/news/international/turkey-syria-earthquake-death-toll-crosses-50000
-feb-25-2023/article66551803.ece

96. Burkina Faso is surrounded by which of the following countries?


1. Mali
2. Niger
3. Nigeria
4. Mauritania

Select the correct answer using the code given below:


A. 1 and 3 only
B. 2, 3 and 4 only
C. 1 and 2 only
D. 1, 2, 3 and 4

Correct Answer : C

instacourses.insightsonindia.com 93

https://upscmaterial.online/
Download From - https://upscmaterial.online/

Total Marks : 200.00


Test-26 (Subject)
( Insta Prelims Test Series 2023 )

Answer Justification :

https://www.thehindu.com/news/international/explained-why-have-french-troops-withdrawn-fro
m-burkina-faso/article66541974.ece

97. Consider the following statements regarding Boards of Directors


1. The President of the World Bank Group serves as Chairman of the Board, and is selected by the
Executive Directors.
2. The Board usually meets twice a week on Tuesdays and Thursdays.
3. The voting power of each Member country is based on the number of shares it holds.

Which of the statements given above is/are correct?


A. 1 only
B. 2 and 3 only
C. 3 only
D. 1, 2 and 3

Correct Answer : D

instacourses.insightsonindia.com 94

https://upscmaterial.online/
Download From - https://upscmaterial.online/

Total Marks : 200.00


Test-26 (Subject)
( Insta Prelims Test Series 2023 )

Answer Justification :

All the statements given above is/are correct.

The Boards of Directors (the Board) is made up of representatives of the Bank’s member
countries that appoint them or elected them. They exercise powers delegated by the Boards of
Governors.

Chairman of the Boards of Directors

The President of the World Bank Group serves as Chairman of the Board, and is
selected by the Executive Directors. The President is the presiding officer, and ordinarily
has no vote except a deciding vote in case of an equally divided Board.

Frequency of Meetings

The Board usually meets twice a week on Tuesdays and Thursdays. Other meetings
such as Board Committee meetings are held at various other times whenever
required.

Voting Powers

The voting power of each Member country is based on the number of shares it holds.
Shares are allocated differently in each organization, resulting in different voting powers.

The Corporate Secretariat is responsible for coordinating the process of membership as well as
assisting members to complete their subscriptions to their allocated shares under periodic
capital increases in IBRD, IDA, IFC, and MIGA. It provides advice on the procedures for
subscribing to additional shares as authorized under resolutions approved by the Boards of
Governors, including required documentation and capital subscriptions payments.

https://www.thehindu.com/news/international/world-bank-board-aims-to-pick-next-president-by-
early-may-us-nominee-expected-soon/article66543282.ece

https://www.worldbank.org/en/about/leadership/directors

98. Pink River Dolphin, sometime seen in the news, can be seen in which of the following
river?

A. Amazon
B. Nile
C. Ganga
D. Mekong

Correct Answer : A

instacourses.insightsonindia.com 95

https://upscmaterial.online/
Download From - https://upscmaterial.online/

Total Marks : 200.00


Test-26 (Subject)
( Insta Prelims Test Series 2023 )

Answer Justification :

The Amazon River dolphin, also known as the pink river dolphin or boto, lives only in
freshwater. It is found throughout much of the Amazon and Orinoco River basins in Bolivia,
Brazil, Colombia, Ecuador, Guyana, Peru, and Venezuela.

https://www.thehindu.com/sci-tech/energy-and-environment/two-endangered-pink-dolphins-resc
ued-from-shallow-colombia-river/article66535461.ece

99. Consider the following statements regarding National Institute of Solar Energy (NISE)
1. NISE is an autonomous specialized institute under the Ministry of New and Renewable Energy
2. NISE is a technical hub for all solar linked activities, standardization, designing, consultancy,
and skill development programs.

Which of the statements given above is/are correct?


A. 1 only
B. 2 only
C. Both 1 and 2
D. Neither 1 nor 2

Correct Answer : C

Answer Justification :

Both the statements are correct.

NISE is an autonomous specialized institute under the Ministry of New and


Renewable Energy (MNRE), Government of India, mandated for research and
development, solar component testing and certification, capacity building, and development of
solar products and applications.

The technical support of NISE complements the requirements of MNRE to become a self-reliable
renewable power producing nation and accept the series of challenges intervened in amidst of
implementation of the National Solar Mission (NSM). NISE has established in the solar energy
sector through continuous efforts by developing newer technologies, developing standards, and
catering to the changing needs in the industry. Furthermore, NISE envisions in accelerating the
proliferation of the renewable energy sector by intently working together with the Government
of India.

NISE is a technical hub for all solar linked activities, standardization, designing,
consultancy, and skill development programs. The institution is committed to deliver
quality in all the efforts put in to promote and grow solar energy in India. The institution aims to
address the gaps and work towards the National Solar Mission in the most significant procedure.

NISE believes on the technology evolutions. This helps to anticipate and prepare for future

instacourses.insightsonindia.com 96

https://upscmaterial.online/
Download From - https://upscmaterial.online/

Total Marks : 200.00


Test-26 (Subject)
( Insta Prelims Test Series 2023 )

challenges. New opportunities for innovation will be the major focus for future growth. Preparing
the solar Industry for a cost optimized solution, with emphasis on quality shall also be included
in the forthcoming plans.

https://nise.res.in/about-us/about-nise/#page-content

100. Greater one-horned rhino, is not naturally found in

A. Pakistan
B. Nepal
C. Bhutan
D. None of the above

Correct Answer : A

Answer Justification :

The greater one-horned rhino once ranged from northern Pakistan, across much of northern
India, Nepal, northern Bangladesh, and Myanmar. It occurred mainly in alluvial grasslands,
where the grass grew up to 8m tall. It was also found in adjacent swamps and forests.

The species came very close to extinction in the 20th century. And by 1975, only 600
individuals survived in the wild in India and Nepal.

Decades of conservation efforts have seen the population grow to 3,500 in the Terai Arc
Landscape of India and Nepal and the grasslands of Assam and north Bengal, northeast India. It
is now found in cultivated areas and pastures, as well as modified woodlands.

With at least half of the total population, India's Kaziranga National Park remains the key
reserve for this species. Now, it is extinct in Pakistan.

https://kuenselonline.com/one-horned-rhinoceros-caught-on-camera-in-royal-manas-national-pa
rk/

https://www.worldwildlife.org/species/greater-one-horned-rhino

instacourses.insightsonindia.com 97

https://upscmaterial.online/

You might also like